Vous êtes sur la page 1sur 84

Risk Management and Insurance Planning

(Workbook)

IMS Learning Resources Pvt Ltd. E-Block, 6th Floor, NCL Bandra Premises, Bandra Kurla Complex, Bandra (E). Mumbai 400 051 Tel No: +91 22 66680005 Fax No: +91 22 66680006 Email: help.fp@imsindia.com Website:www.imsindia.com

PREFACE

Having gained conceptual clarity through concept book experience, it is important that you apply and strengthen your fundamentals; the questions in this workbook will offer you practice and challenge your ability and understanding. The questions have been designed in such a way that they cover all the concepts discussed in the concept book. The questions in this workbook aims to help you identify your conceptual gap and assess your strengths and areas for improvement. If you need any support in your preparation; Do write to us at help.fp@imsindia.com In addition to these workbooks, you can also access more practice tests online. For details write to us at help.fp@imsindia.com

Risk Management and Insurance Planning

Workbook

1.

Composite Insurance Agent holds a license to act as an Insurance agent for a ______ a) b) c) d) Life Insurer General Insurer Both of the above None of the Above

7.

_______________ is the right of one person, having indemnified another under a legal obligation to do so, to stand in the place of the other and avail himself of all the right and remedies of that other, whether already enforced or not. a) b) c) d) Substitution Indemnity Adhesion Subrogation

2.

In property insurance (excluding marine insurance) insurable interest must exist at the time of a) b) c) d) Loss Contract Both (a) & (b) None of the above 8.

Touching another person unlawfully is an example of ___________ a) b) c) d) Intentional Tort. Absolute liability. Negligence. Conclusive tort.

3.

Gambling is an example of _______. a) b) c) d) Pure risk Speculative risk Personal risk Property risk 9.

Pricing of a product in a Life insurance company is a function of _____ department. a) b) c) d) Marketing Underwriting Actuarial Finance

4.

Which of the following is the third step in identifying risks? a) Identify risks b) Develop Alternative for handling risks c) Choose and implement an appropriate Strategy d) Analyze and evaluate risks (Risk Measurements) 10.

Collection and payment of monies in Life insurance company is a function of _____ department. a) b) c) d) Marketing Actuarial Accounting and Finance Legal and Secretarial

5.

Pure risk can be defined as a situation where: a) b) c) d) there is a chance of loss or no loss there is a chance of a loss or a profit there is a no chance of loss all of the above 11.

Internal Auditing in Life insurance company is a function of _______ department. a) b) c) d) Underwriting Actuarial Finance Legal and Secretarial

6.

An organization cannot finance the cost of its own risk by ______________. a) b) c) d) paying out of current expenses debt or equity financing pre or post credit approaching a reinsurance company 12.

Compliance to regulator and other public supervisory bodies in Life insurance company is a function of ____________ department. a) b) c) d) Underwriting Actuarial Finance Legal and Secretarial

Workbook

Risk Management and Insurance Planning

13.

The amount of insurance accepted by the re-insurer is known as ________ a) b) c) d) Premium Net premium Cession Retention

19.

The limit of indemnity for a Direct Broker is a) Three times remuneration received at the end of every financial year subject to a minimum limit of rupees fifty lakhs. b) Three times remuneration received at the end of every financial year subject to a minimum limit of rupees two crores and fifty lakhs. c) Three times remuneration received at the end of every financial year subject to a minimum limit of rupees five crores. d) Three times remuneration received at the end of every financial year subject to a minimum limit of rupees ten crores.

14.

Which amongst these policies meet the need of security for childrens education the best? a) b) c) d) Term assurance plan Children deferred assurance plan Deferred annuity plan Health insurance 20.

15.

Insurer has to decide about a claim within ________ days of submission of all reports. a) b) c) d) 30 days 45 days 60 days 90 days

An ultimate mortality table excludes data for: a) b) c) d) first 5 to 15 years first 3 to 5 years first 3 to 6 years first 7 to 15 years

21.

16.

For any applicant to be registered as an IRDA licensed agent, the minimum qualification required in an urban area is _________. a) b) c) d) X th standard XII th standard Graduation Post-Graduation

The cheapest policy amongst the following policies in the market is ____________ a) b) c) d) A term policy A unit linked policy An endowment policy A Whole life policy

22.

Participating polices are policies entitled to _________. a) b) c) d) risk sharing premium bonus fixed payments

17.

Employees liability for his employees injury even when there is no negligence on the part of the employer is an example of ____. a) b) c) d) Intentional Tort Absolute liability Negligence Conclusive tort

23.

An increasing term insurance means _____ a) The term insurance sum assured increases with inflation b) Both the premium and sum assured increases c) The term insurance sum assured increases at intervals d) Only the premium keeps on increasing

18.

Investigation regarding mortality factor in an Life insurance company is a function of _______ department. a) b) c) d) Marketing Underwriting Actuarial Legal and secretarial

24.

Overseas mediclaim policy does not cover one of the following: a) b) c) d) Loss of passport Loss of baggage Medical accidents Personal accident

Risk Management and Insurance Planning

Workbook

25.

The process of due diligence conducted by an insurance agent is known as _______ a) b) c) d) Underwriting Investigation Inspection Site Check

32.

The limit of indemnity for a Reinsurance Broker is a) Three times remuneration received at the end of every financial year subject to a minimum limit of rupees fifty lakhs. b) Three times remuneration received at the end of every financial year subject to a minimum limit of rupees two crores and fifty lakhs. c) Three times remuneration received at the end of every financial year subject to a minimum limit of rupees five crores. d) Three times remuneration received at the end of every financial year subject to a minimum limit of rupees ten crores.

26.

Which type of insurance is not available in India? a) b) c) d) Officers liability Disability Income Protection. Health Life

27.

Insurance Advisors and Agents are governed by: a) b) c) d) IRDA SEBI AMFI Both IRDA & SEBI

33.

Which policy meets the need of Security after retirement the best a) b) c) d) Term assurance plan Children deferred assurance plan Deferred annuity plan Health insurance.

28.

Mortality rate is : a) b) c) d) Rate of death in a group Rate of death in the population Probability of death in a group Probability that the insured will die before reaching a certain age 34.

A sub standard Indian life is _________ a) b) c) d) an individual with health problems below average intelligent Indian life a life which cannot be insured dependent on the underwriting standards of the insurance company

29.

HLV explains in general : a) b) c) d) Total income of an individual Total worth of an individual Total wealth of an individual Present value of Earning Capacity 35.

In insurance, the contingency insured against is usually the ___________ a) b) c) d) Financial loss Certain financial loss Loss of profit Uncertain financial loss

30.

Adverse loss ratio refers to : a) Higher premium being received by the insurer b) Lower premium being received by the insurer c) Lower than expected claim ratio in the pool d) Higher than expected claims ratio in the pool 36.

Insurance companies often offer the services of specialists and experts to industry in different fields. This is for rendering advice on ___________ a) b) c) d) Reduction in uncertainty Loss prevention/minimization Indemnification None of the above

31.

The other name of no fault liability is _____ a) b) c) d) Absolute Liability Strict liability Vicarious Liability Contractual Liability

Workbook

Risk Management and Insurance Planning

37.

The limit of indemnity for a Composite Broker is a) Three times remuneration received at the end of every financial year subject to a minimum limit of rupees fifty lakhs. b) Three times remuneration received at the end of every financial year subject to a minimum limit of rupees two crores and fifty lakhs. c) Three times remuneration received at the end of every financial year subject to a minimum limit of rupees five crores. d) Three times remuneration received at the end of every financial year subject to a minimum limit of rupees ten crores.

43.

Health insurance for children is provided for children aged 4 years __________. a) With no precondition b) When one of the parents is being insured c) When both the parents are covered concurrently d) Both (a) and (b)

44.

Domicillary hospitalization period is defined as _________ a) b) c) d) One day Two days Three days Four days

38.

The disadvantage of a term policy is : a) b) c) d) it is expensive it is only for a short period there is no bonus All the above

45.

Jan Arogya Bima policy is a __________. a) b) c) d) Life insurance Policy Marine insurance policy Travel insurance policy Health insurance

39.

Market Research of existing products in an Life Insurance Company is a function of ______ department. a) b) c) d) Marketing Underwriting Actuarial Finance

46.

Publication of a false statement in a permanent form designed to damage the reputation of another person is an example of____________ a) b) c) d) Intentional Tort Absolute liability Negligence Conclusive tort

40.

The principle of subrogation is derived form the principle of : a) b) c) d) Utmost good faith Adhesion Duty of disclosure Indemnity

47.

Bhavishya Arogya policy is meant for persons in the age group between _______ a) b) c) d) 25 30 25 30 to 55 years to 55 years to 60 years to 60 years

41.

The duty of disclosure follows from : a) b) c) d) The The The The doctrine of adhesion principal of subrogation principle of indemnity principle of utmost good faith 48.

The disadvantage of a whole life policy is : a) b) c) d) it is expensive it is only for a short period there is no bonus All of the above

42.

Offer in life insurance contract refers to : a) Proposer paying first premium b) Proposers filling up application form for taking insurance c) Original policy document d) Companys brochure duly authenticated 49.

In insurance parlance TPA means: a) b) c) d) Third Party Assessor Third Party Adjustor Third Party Administrator Total Premium Amount

Risk Management and Insurance Planning

Workbook

50.

The working capital of a TPA should be minimum a) b) c) d) Rs. 5,00,000 Rs. 10,00,000 Rs. 5,00,000 Rs. 10,000,000

56.

Particular risk is a risk that affects : a) b) c) d) Entire economy Only individual Government None of the above

57.

51.

Commercial vehicle insurance does not include : a) b) c) d) Goods carrying vehicle insurance Passenger carrying vehicle insurance A family car insurance All of the above 58.

Home loan/Mortgage protection insurance refers to _________ a) Property insurance policy b) Credit insurance policy c) Life insurance policy guaranteeing the mortgage loan d) None of the above The most basic risk management decision is __________. a) Whether to provide fire protection equipment b) Perhaps to explore, whether to purchase insurance c) Analyze the probability distribution d) None of the above

52.

In life insurance, insurable interest must exist at the time of a) b) c) d) Loss Contract Both (a) & (b) None of the above

53.

The insurance company has to respond to all communication received from the policyholders within : a) A week of receipt of such communication b) 15 days of their receipt c) 10 days of their receipt d) 20 days of their receipt

59.

It is compulsory for the broker to maintain _______. a) b) c) d) Liquidity margin Cash margin Solvency margin Cash balance

54.

In hazard logic tree, under the peril water damage, hazards are __________ a) Malicious Damage, subsidence, hurricane b) Flood, storm, bust pipe or other apparatus, burst water main c) Spillage or leakage or Flammable Liquids d) Subsidence and flooding

60.

To keep the license in force the composite broker shall have to pay registration fees calculated at the rate of ________ per annum. a) b) c) d) Rs. 2 Lakhs Rs. 3 Lakhs Rs. 25,000 Rs. 50,000

61.

Quick asset ratio is used to: a) Determine the investment capacity of the firm b) Determine firms liquidity to cover unexpected payment. c) Determine the current assets of the firm d) Determine the current liabilities of the firm

55.

Which of the following is not a classification of term insurance policy? a) sum assured with bonus return term insurance b) Level benefit term assurance c) Increasing benefit term assurance d) Decreasing benefit term insurance

62.

Standard deviation and coefficient of variation are: a) b) c) d) used in fair value accounting economic indicators methods used to measure dispersions none of the above

Workbook

Risk Management and Insurance Planning

63.

Product development in an Life insurance company is a function of ______ department a) b) c) d) Marketing Underwriting Actuarial Finance

69.

An Indian Insurance company can transact in __________ a) Life insurance and general insurance business b) Life insurance and reinsurance business c) General and reinsurance business d) Either Life or general insurance or reinsurance business

64.

Self insurance funds may be identified as: a) Contingency funds used to spread the cost of losses more evenly over a large term b) Profitability not due to normal trading, but because of exceptionally good or bad loss experience c) Contribution collected from each of the division of the company d) None of above 70.

The reinsurance contract itself may relate to a) b) c) d) One particular company One particular ceding insurer One particular insurance category One particular catastrophe

71.

The following factor is least important in the view of an underwriters _____ a) b) c) d) Age Sex Occupation Family income

65.

If substandard risk is accepted : a) This may lead to bankruptcy of the Insurance co b) Outcome will be that of a speculative risk c) Outcome will be that of a dynamic risk d) None of the above

72.

In fire insurance, a reinsurance policy is called as __________ a) A guarantee policy but nowadays is more commonly referred to as facultative reinsurance. b) A coagulative policy on property insurances with specific focus on catastrophe insurances. c) A fire fighting comprehensive policy with focus on mega risks. d) An extraneous perils covering policy in the event of disaster.

66.

Charging of losses to current operating costs a) b) c) d) Is a form of Is a form of Is a form of Is a form of reducing losses evaluating the losses financing the losses Identifying the losses

67.

The waiting period under a disability benefit policy refers to ___________ a) The period of time that must elapse before the policy benefit commence. b) The period of time that must elapse between submission of claim and settlement of the claim. c) The period of time that must elapse before the issue of the policy document. d) The period of time that must elapse before the proposal can be considered. 73.

Ram insures his home worth Rs. 50 Lakhs for Rs. 30 Lakhs. The house is destroyed in a fire and he suffers losses worth Rs. 20 Lakhs. How much will he receive from the insurance company? a) b) c) d) Rs. Rs. Rs. Rs. 10 Lakhs 20 Lakhs 16 Lakhs 12 Lakhs

68.

The law of reinsurance is primarily based on a) b) c) d) law of torts law of vicarious liability law of contract law of specific relief

74.

Reinsurance cannot exist____________ a) b) c) d) Without its global character Without captive insurers Without there being direct insurance. Without past data of catastrophe risk covered

Risk Management and Insurance Planning

Workbook

75.

The application of the law of contract does not apply to _____________contracts. a) b) c) d) Insurance Stock markets Property deals None of them

81.

76.

Every reinsurance instrument executed out of India chargeable with duty may be stamped __________ a) b) c) d) Within four months Within five months Within three months Within six months

Policy cost/conversion = 0.001, Interest allowed equivalent to after tax return =19%, annual premium = Rs. 14,000, dividend or bonus = Rs. 22,000, death benefits = Rs. 25,00,000, cash surrender value at the end of current policy period = Rs. 6,00,000, cash surrender value at the end of previous policy year = Rs. 580000. What is policy cost per thousand? a) b) c) d) Rs. 19.70 Rs. 44.66 Rs. 57.84 Rs. 122.51

82.

77.

Insurance can be ________contracts. A = Benefit, B= Indemnity, C= Negotiated. a) b) c) d) Only A Either B or C Only A Only B

Policy cost/conversion = 0.001, Interest allowed equivalent to after tax return =18%, annual premium = Rs. 14,000, dividend or bonus = Rs. 21,000, death benefits = Rs. 30,00,000, cash surrender value at the end of current policy period = Rs. 5,90,000, cash surrender value at the end of previous policy year = Rs. 550000. What is policy cost per thousand? a) b) c) d) 19.70 44.66 22.62 122.51

78.

Fundamental risk is a risk that affects: a) b) c) d) Entire economy Only individual Government None of the above

83.

79.

Which one of the following is not a personal risk? a) b) c) d) risk of risk of risk of risk of premature death sickness unemployment car accident

Policy cost/conversion = 0.001, Interest allowed equivalent to after tax return =18% annual premium = 15000, dividend or bonus = 20000, death benefits = 20,00,000, cash surrender value at the end of current policy period=600,000, cash surrender value at the end of previous policy year = 525,000. What is policy cost per thousand? a) b) c) d) 19.70 44.66 57.84 12.29

80.

Policy cost/conversion = 0.001, Interest allowed equivalent to after tax return =20%, annual premium = Rs. 13,000, dividend or bonus = Rs. 23,000, death benefits = Rs. 20,00,000, cash surrender value at the end of current policy period = Rs. 6,50,000, cash surrender value at the end of previous policy year = Rs. 570000. What is policy cost per thousand? a) b) c) d) 19.70 44.66 57.84 122.51

84.

Policy cost/conversion = 0.002, Interest allowed equivalent to after tax return =20%, annual premium = Rs. 13,000, dividend or bonus = Rs. 23,000, death benefits = Rs. 20,00,000, cash surrender value at the end of current policy period = Rs. 6,50,000, cash surrender value at the end of previous policy year = Rs. 570000. What is policy cost per thousand? a) b) c) d) 9.85 22.33 28.92 61.25

Workbook

Risk Management and Insurance Planning

85.

Policy cost/conversion = 0.002, Interest allowed equivalent to after tax return =19%, annual premium = Rs. 14,000, dividend or bonus = Rs. 22,000, death benefits = Rs. 25,00,000, cash surrender value at the end of current policy period = Rs. 6,00,000, cash surrender value at the end of previous policy year = Rs. 580000. What is policy cost per thousand? a) b) c) d) 9.85 22.33 28.92 61.25

89.

A group of 75000 persons each aged 33 years wish to apply for term insurance for a one year period for a sum of Rs.2,50,000. If mortality tables show that out of 100,00,000 people 25,000 die within a year, find the premium to be paid by each of the 75000 applicants. a) b) c) d) 625 700 750 800

90.

86.

Policy cost/conversion = 0.002, Interest allowed equivalent to after tax return =18%, annual premium = Rs. 14,000, dividend or bonus = Rs. 21,000, death benefits = Rs. 30,00,000, cash surrender value at the end of current policy period = Rs.5,90,000, cash surrender value at the end of previous policy year = Rs. 550000. What is policy cost per thousand? a) b) c) d) 9.85 22.33 11.31 61.25 91.

A group of 45000 persons each aged 34 years wish to apply for term insurance for a one year period for a sum of Rs. 3,50,000. If mortality tables show that out of 60,00,000 people 30,000 die within a year, find the premium to be paid by each of the 45000 applicants. a) b) c) d) 1650 1700 1750 1800

87.

Policy cost/conversion = 0.002, Interest allowed equivalent to after tax return =18% annual premium = Rs. 15000, dividend or bonus = Rs. 20000, death benefits = Rs. 20,00,000, cash surrender value at the end of current policy period = Rs. 600,000, cash surrender value at the end of previous policy year = Rs. 525,000. What is policy cost per thousand? a) b) c) d) 9.85 22.33 28.92 6.14

A group of 55000 persons each aged 40 years wish to apply for term insurance for a one year period for a sum of Rs. 3,00,000. If mortality tables show that out of 75,00,000 people 50,000 die within a year, find the premium to be paid by each of the 55000 applicants. a) b) c) d) 1800 1900 2000 2100

92.

88.

A group of 50000 persons each aged 35 years wish to apply for term insurance for a one year period for a sum of Rs. 2,00,000. If mortality tables show that out of 50,00,000 people 30,000 die within a year, find the premium to be paid by each of the 50000 applicants. a) b) c) d) 1200 1250 1300 1400

A group of 35000 persons each aged 30 years wish to apply for term insurance for a one year period for a sum of Rs. 3,00,000. If mortality tables show that out of 120,00,000 people 60,000 die within a year, find the premium to be paid by each of the 35000 applicants. a) b) c) d) 1500 1250 1400 2750

10

Risk Management and Insurance Planning

Workbook

93.

A group of 50000 persons each aged 28 years wish to apply for term insurance for a one year period for a sum of Rs. 2,50,000. If mortality tables show that out of 50,00,000 people 25,000 die within a year, find the premium to be paid by each of the 50000 applicants. a) b) c) d) 1500 1250 1400 2750

97.

94.

A group of 60000 persons each aged 45 years wish to apply for term insurance for a one year period for a sum of Rs. 2,80,000. If mortality tables show that out of 80,00,000 people 40,000 die within a year, find the premium to be paid by each of the 60000 applicants. a) b) c) d) 1500 1250 1400 2750

Calculate the HLV of Mr. Apte. His present age 45 years. His retirement age is 70 yrs. His total income of Rs. 21,00,000 (Salaries 15 Lakhs, dividends on individual shares 2.5 Lakhs, HUF co-partner profit share 1.5 Lakhs, interest on bank deposit and other investment 1 Lakh, minor daughters income added 1 Lakh.) he has paid following taxes: Corporate professional tax 5000 income tax 410000 as per individual tax return filed. He pays total life insurance premium of Rs. 55000 (self insurance cover S.A. 22,00,000) Reasonable maintenance charge for a person of his stature is assumed as Rs. 100000 p.a. applied rate of interest to arrive at a present value of his future income is at 5% a) b) c) d) Rs. 2.26 crores Rs. 3 crores Rs. 1.31 crores Rs. 12 crores

98.

95.

A group of 55000 persons each aged 40 years wish to apply for term insurance for a one year period for a sum of Rs. 3,00,000. If mortality tables show that out of 60,00,000 people 55,000 die within a year, find the premium to be paid by each of the 55000 applicants. a) b) c) d) 1500 1250 1400 2750

A planner and prospect agreed for an endowment policy for Rs. 4 Lakhs limited payment for 10 years with a term of 20 years. If reversionary bonus is taken as 7.5% per annum and terminal bonus as Rs. 150 per 1000 what will be maturity value? a) b) c) d) Rs. 10,50,000 Rs. 10,60,000 Rs. 10,70,000 Rs. 10,80,000

99.

Which one of the following is not a risk faced by the individual in their various activities? a) b) c) d) Financial risk Cumulative risk Pure risk Speculative risk

96.

Calculate HLV to recommend adequate insurance cover; Mr. Ritesh, aged 30 yrs, retirement age 60 years. He is Asst. Vice President. His monthly salary is 55,000. he pays professional tax of Rs. 3000 and income tax of Rs. 132000 Reasonable self maintenance expenditure estimated Rs. 45000 p.a.; life insurance premium for self Rs. 18000 with total sum assured Rs.1200000. For wife and child he pays insurance premium of Rs. 10500 and Rs.6500 respectively rate of interest assumed for capit alization of f uture income is at 8%. Adequate additional insurance recommended is a) b) c) d) Rs. 50,00,000 Rs. 40,00,000 Rs. 55,00,000 Rs. 10,000,000.

100. A person goes to the neighborhood lottery store and buys a lottery ticket. This is an example of _________. a) b) c) d) Pure risk Dynamic risk Speculative risk Non financial risk

101. A living person will face the risk of death at all points of their life and this risk will be classified as a ___________ a) b) c) d) Personal risk Property risk Liability risk None of the above

Workbook

Risk Management and Insurance Planning

11

102. Every time the children move out to play in the afternoon in the summer holidays their mother insists that they wear a cap to keep out the impact of the direct rays of the sun and to drink water to avoid dehydration. This is an example of ____________ a) b) c) d) Risk transfer Risk avoidance Risk retention Risk control

108. The role of insurance in the economy will relate to the process of ___________ a) Providing funds for investment and credit b) Controlling inflation c) Pushing up interest rates d) Controlling the foreign exchange 109. A person wants insurance for his motorcar. The point regarding the colour of the car will be a ______ point for the purpose of insurance a) b) c) d) Material Slightly material Non material None of the above

103. A footballer who is voted as the European Footballer of the year does not take an insurance policy on his legs and continues to play as before. This is a case of _______ a) b) c) d) Risk transfer Risk avoidance Risk retention Risk reduction

110. Under the principle of insurable interest there should be _______ that can be insured a) b) c) d) Property Life Potential liability All of the above

104. While deciding upon the risks to be insured the various factors considered for the process are _________ a) b) c) d) Various events of risk Severity of the financial loss Frequency of occurrence All of the above

111. Which of the following is not a factor covered for being insured under the principle of insurable interest? a) b) c) d) Right Space Interest Limb

105. All the risks present for a person can be insured. The statement is ____________ a) True, because risk can be transferred b) False, because it has to fulfill the required conditions of measurability, homogeneity etc c) True, because a person can take the necessary insurance d) False, because everyone does not have all the different kinds of risk 106. An insurance contract can be described as a contract of ___________. a) b) c) d) Indemnity Profit Capital gains Earning return

112. The insured must have a _______ with the subject matter of insurance in case they are seeking an indemnification. a) b) c) d) Right Interest Relationship None of the above

113. A person owns a building in which he stays. In this case insurable interest is created by: a) b) c) d) Nature Common law Contract Statute

107. Insurance helps a person to ________ a) b) c) d) Prevent a profit Prevent a loss Make a capital gain Earn a rate of return

114. A person has ______ insurable interest in his own life. a) b) c) d) part specific unlimited limited

12

Risk Management and Insurance Planning

Workbook

115. A person got married in the last week of December 2005. In the month of November 2006 does the wife have an insurable interest in the life of the husband? a) No, because a year has not passed since marriage b) Yes, because a person has an automatic insurable interest in the life of the spouse c) No, because only the husband has an insurable interest in the life of the wife d) None of the above 116. In case of a mortgage the situation for the two parties in terms of insurable interest is: a) Only the mortgagor has insurable interest b) Only the mortgagee has insurable interest c) Both have insurable interest d. None have insurable interest 117. _________ will have insurable interest on the subject in the role that they play a) b) c) d) Trustees Bailee Executors All of the above

121. A life insurance policy states that the death on account of a war will not be covered by the insurance policy. This will be classified as a __________ a) b) c) d) Uninsured peril Insured peril Excluded peril None of the above

122. In an insurance contract the side making the offer is ___________ a) b) c) d) The insurance company The insured The beneficiary All of the above

123. The consideration in the insurance contract is the payment of _________ a) b) c) d) Commission Premium Underwriting amount Fixed amount of payment

124. The value exchanged in terms of premium paid and the payout in case of a loss is equivalent. That is why Insurance contract is also known as ___________. a) b) c) d) Aleatory contract Fixed contract Adhesion Utmost good faith

118. An unsured is not supposed to profit from an insurance policy but should be in a position similar to what he was in before the loss occurred. This is outlined by the principle of: a) b) c) d) Utmost good faith Insurable Interest Indemnity Subrogation

125. If an insurance company looks at the specific case and determines premium rate, it is an example of _________ a) b) c) d) Individual rating Class rating Merit rating None of the above

119. The feature of indemnity can be undertaken through the route of _________ a) b) c) d) Repairs Reinstatement Replacement All of the above

126. _________ is not a way of category of merit rating for an insurance company a) b) c) d) Schedule rating Experience rating Class rating Retrospective rating

120. The risk that are covered by an insurance policy are called ___________ a) b) c) d) Uninsured perils Insured perils Excluded perils None of the above

Workbook

Risk Management and Insurance Planning

13

127. All of the following statements describe the operation of a life annuity EXCEPT: a) Because of the interest factor, an annuitant is assured of receiving back more than he or she paid in b) The annuitant is assured that he or she cannot outlive the length of time of the annuity payments c) The emphasis is on the liquidation of the fund as opposed to its growth d) The older the annuitant is when he or she receives the first annuity payment; the greater will be the amount of each payment 128. A type of risk with high frequency but low severity is probably best handled by ______ a) b) c) d) Avoidance Subrogation Self-insurance Under-Insurance

132. In Unit-Link policy, market risk is with ______. (A) Insured (B) Insurer a) b) c) d) A B Both A & B Neither A nor B

133. Minimum age at entry for Medi-claim is ________. a) b) c) d) 8 years 91 days 365 days 18 years

134. An insurance company pays the insured the sum for the goods destroyed in his godown. There is still some salvage in the godown and the value would be around of Rs. 5000. In this situation the share of the insurance company is : a) b) c) d) Rs. 0 Rs. 2500 Rs. 5000 Rs. 10000

129. Speculative risk can have following outcomes _______. (A) a) b) c) d) Loss (B) Gain Only A Only B Only C A, B and C (C) Status Quo

135. There is a godown that is worth Rs. 20 Lakhs. A person insures it for Rs. 15 Lakhs and there is a fire that completely destroys the godown. In this case the insurance company will pay : a) b) c) d) Rs. Rs. Rs. Rs. 500000 1000000 1125000 1500000

130. LALGI is ___________. a) Private contribution guarantee scheme b) Private insurance c) Public benefit guarantee scheme d) Social insurance 131. For risk to be insurable which one of following is not correct? a. The loss must be fortuitous or accidental. b. The loss must not be catastrophic. c. The loss produced by the risk must be definite and measurable. d. There must be a sufficiently large number of heterogeneous exposure units to make the losses reasonably predictable.

136. A property is worth Rs. 10 Lakhs and an individual insures it for a sum of Rs. 12 Lakhs. In this case when a fire destroys the full property then the insurance company will pay the insured a sum of _________ a) b) c) d) Rs. Rs. Rs. Rs. 500000 800000 1000000 1200000

137. There is an asset worth Rs. 10 Lakhs and this is insured for a sum of Rs. 8 Lakhs. There is a loss wherein half of the asset is destroyed. The insurance company will pay a sum of : a) b) c) d) Rs. Rs. Rs. Rs. 320000 400000 500000 800000
Workbook

14

Risk Management and Insurance Planning

138. There is an excess of 10% in a policy and in the example where there is an amount of Rs. 20000 that will be the claim then the insurance company will pay _________ a) b) c) d) Rs. 0 Rs. 2000 Rs. 18000 Rs. 20000

amount that will be paid by the insurance companies together is : a) b) c) d) Rs. Rs. Rs. Rs. 1000000 2000000 2500000 4000000

139. There is an excess of Rs. 5000 in a policy and the amount spent for the expense that has to be reimbursed under the policy comes to Rs. 4500. The insurance company will pay _______. a) b) c) d) Rs. 0 Rs. 500 Rs. 4500 Rs. 5000

144. A person takes insurance for Rs. 5 Lakhs from one insurance company and Rs. 3 Lakhs from another for the asset worth Rs. 8 Lakhs. In this case when there is a loss of Rs. 6 Lakhs then the total sum paid by the two insurance companies put together will be _______ a) b) c) d) Rs. 5 Lakhs Rs. 6 Lakhs Rs. 8 Lakhs None of the above

140. A property insurance policy has an excess of Rs. 5000 and a sum assured of Rs. 1 Lakh. When there is a loss of Rs. 55000 then the insurance company will pay a sum of _____ a) b) c) d) Rs. 0 Rs. 5000 Rs. 50000 Rs. 55000

145. If the rate of insurance is Rs. 3 per thousand per year and there is an insurance policy of Rs. 3 Lakhs then the premium will come to _______ a) b) c) d) Rs. Rs. Rs. Rs. 300 600 900 990

141. A policy has a limit of Rs. 1 Lakh. When there is a claim of Rs. 200000 by the person insured then the insurance company will pay a sum of ________ a) b) c) d) Rs. 0 Rs. 50000 Rs. 100000 Rs. 200000

146. The following figure is not a premium figure paid by a holder of an insurance policy : a) b) c) d) Rs. Rs. Rs. Rs. 2500 1345 3546 1.2 per thousand

142. There is an asset worth Rs. 10 Lakhs. A person insures this for Rs. 5 Lakhs each with 2 different insurance companies. If the ent ire asset is destroyed t hen each insurance company will pay __________ a) b) c) d) Rs. Rs. Rs. Rs. 250000 500000 750000 1000000

147. An insurance company has been in the business of insurance in particular country since the last 15 years. The company actively uses the data that has been generated for the past several years for the purpose of developing the insurance rates that it will charge its customers. This process is known as : a) b) c) d) Insurance coverage Loading Premium process None of the above

143. There is an asset worth Rs. 20 Lakhs. A person insures this for Rs. 20 Lakhs each with two insurance companies. If the asset is destroyed to the extent of 50% then the

Workbook

Risk Management and Insurance Planning

15

148. Premium on Motor Insurance Policy does not depend on which one of the following factors? a) Zone of operation of the vehicle b) Insured Declared Value (IDV) of the vehicle c) Cubic capacity of the vehicle d) Age of the owner of the vehicle 149. In several unit linked polices the _____ charge can be as high as 30-35% a) b) c) d) First year charge Administrative charge Risk charges Asset management fees

153. When a rider is taken on an insurance policy along with an additional benefit there would be an additional cost in terms of _______ a) b) c) d) Higher premium Lower premium Lower benefit Same benefit

150. A person took a life insurance policy on 2 October 2005. 25 days later he slashed his wrists after an argument with his family and consequently died. The claim under the policy will ___________ a) be paid because the policy is in his name b) be paid because the policy has completed 15 days c) not be paid because he did not die due to a disease d) not be paid because of coverage under the suicide clause 151. A person has a premium due on 15 November. Since he was traveling abroad he has not been able to pay the premium. After he is back on 25 November can he pay the premium? a) Yes, because a person can pay premium anytime b) Yes, because the time falls within the grace period clause of the policy c) No, because the due date was the last date for payment d) No, because the premium can only be paid after a month 152. A person wants to ensure that while he has taken the necessary life insurance cover there is also some protection to him and his family in case there is a serious illness that affects the person, which can drain him of financial resources. In such a situation one can take the option of which rider? a) b) c) d) Waiver of premium Accidental death Critical illness Guaranteed insurability

154. A person has a health insurance policy. He had to be admitted to a hospital for the operation and after this was completed he paid the bill and forwarded the insurance claim to his insurance company. After making the necessary checks the insurance company paid the amount. This is an example of a __________ a) b) c) d) Indemnity plan Managed care plan Disability insurance Accident insurance

155. A person owns a large house, which is lent out to tourists as a hotel. During the rains the road leading to the property was washed away leading to loss of income as the tourists were not able to come to the property. This is an example of : a) b) c) d) Direct property risk Indirect property risk Loss of income All of the above

156. A person enters into a contract in the course of his printing business. Due to non fulfillment of the contract he was required to pay damages to some other party. This is an example of : a) b) c) d) Statutory liability Personal liability Professional liability All of the above

157. The overflowing of the river led to floods in Surat this year. After being trapped in their houses for over 5 days the Soni family was able to have access to food and water. In order to tackle such a situation in the future the family moved to Jaipur. This action of the family tackling the risk will be classified as _______. a) b) c) d) Risk transfer Risk avoidance Risk retention Risk reduction

16

Risk Management and Insurance Planning

Workbook

158. A person has a fire in his godown and goods to the extent of Rs. 235000 are destroyed in the fire. There is no other loss because of the fire and the person after looking at the situation files a claim for Rs. 275000. This is an example of ________ a) b) c) d) Normal claim Fraudulent claim Inflated claim None of the above

163. A person has a share of 50% in a property with his partner. The property is valued at Rs. 20 Lakhs. If the person takes an insurance cover on full value of the property, he will be considered as a deemed agent for the cover of _________ a) b) c) d) Rs. 0 Rs. 1000000 Rs. 1500000 Rs. 2000000

159. A painting is insured with a private collector for Rs. 25 Lakhs. He quietly ships off the painting to one of his associates and then claims the insurance with the company for the required sum stating that the painting was stolen while he was on vacation. This is an example of _________ a) b) c) d) Normal claim Fraudulent claim Inflated claim None of the above

164. Sharad own various properties in the city of Jaipur. He has an agent Jayesh who operates and undertakes several activities on his behalf. Jayesh goes to an insurance company to take insurance on the property on behalf of Sharad. In this case there ____________. a) Is no insurable interest b) It is upto the insurance company to decide on the matter c) There is an insurable interest for the agent on behalf of the principal d) None of the above 165. A person got married in the last week of December 2005. In the month of April 2006 the husband wants to know whether he has insurable interest in the property of his wife? a) No, he has insurable interest only in his own property b) No, there cannot be any question of insurable interest for a year after marriage c) Yes, there is insurable interest because the marriage is not registered d) Yes, because spouses have insurable interest in each others property 166. Vinayak, 36 years and married, works for a multinational firm, which provides adequate medical and related covers. He is also able to accumulate sick leave. He already owns a house and has savings of Rs. 35 Lakhs, which are well invested. Which insurance cover does he require the most? a) b) c) d) Life Cover Medical Cover Property Insurance Temporary Total Disablement Cover

160. A person gives out his motorcycle on lease to his friend with the condition that any tax or repairs that will arise on the vehicle will have to be borne by the user. In this case the user has an insurable interest because a) b) c) d) There is no insurable interest Common law Contract Statue

161. A person has borrowed Rs. 2 Lakhs from his friend. Out of this a sum of Rs. 50000 has already been paid back. The amount of insurable interest of the person who had loaned the money in the life of the borrower is _________ a) b) c) d) Rs. 0 Rs. 50000 Rs. 150000 Rs. 200000

162. A person has a share of 50% in a property with his partner. The property is valued at Rs. 20 Lakhs. The person is allowed to take an insurance cover on the policy to the extent of __________. a) b) c) d) Rs. 0 Rs. 1000000 Rs. 2000000 None of the above

Workbook

Risk Management and Insurance Planning

17

167. __________ Insurance protects oneself against losses as a result of lawsuits. a) b) c) d) Life Medical Professional Indemnity Home

168. Loss of income of earning member of family is a ___________ type of risk. a) b) c) d) Liability Personal Property Professional

173. Kavita purchased a collision insurance on her new car. While Kavita was driving home from work, another driver failed to stop at the traffic signal and hit Kavitas car. Kavita called up her insurance agent and reported the accident. The agent said, Dont worry, Kavita, well pay to get your car fixed. And after we pay for the damage to your car, we will try to collect the damages from the driver who damaged your car. The process the agent described is called __________. a) b) c) d) Waiver. Consideration. Estoppels. Subrogation.

169. Only a ______ insurance policy can be assigned a) b) c) d) Health Life Property Liability

174. When someone reasonably relies upon a representation of fact, what legal doctrine prevents the representation of fact from being retracted if the individual who relied upon the representation of fact would be harmed? a) b) c) d) Estoppel Warranty Waiver Agency

170. General insurance policies shall be void unless the person affecting the insurance has an insurable interest in that property at the time the policy is affected as well as at the time of the occurrence of the insured event. a) True b) False 171. If the life insurance policy is endorsed under MWP Act, 1874 then _________________. a) Creditors have first claim over policy proceeds. b) Creditors have no claim over policy proceeds. c) Creditors have residual claim over policy proceeds. d) Income tax authorities have claim over policy proceeds. 172. Which of the following statements about law and the insurance agent is (are) true? i. An insurer is responsible for the acts of its agents when the agents are acting within the scope of the agency agreement. ii. Consumers should not assume that an agency relationship exists just because someone claims to represent an insurance company. a) b) c) d) I only II only Neither I nor II Both I and II

175. The assignment of property insurance by the seller of the property to the purchaser of the property is only valid if the insurer approves the assignment. The reason that the insurer must approve the assignment of a property insurance policy is that: a) Insurance contracts are contracts of utmost good faith. b) Insurance contracts are bilateral contracts. c) Insurance contracts are personal contracts. d) Insurance contracts are aleatory contracts. 176. When Mira applied for a life insurance policy, she answered No in response to the question Have you visited a doctor for any reason during the previous 12 months? In fact, Mira visited a doctor five weeks ago after experiencing chest pain. She was referred to a specialist who determined that Mira has severe heart disease. If she dies shortly after the life insurance policy is issued, upon what grounds will the insurer be successful in denying the claim? a) b) c) d) Concealment Waiver Warranty Misrepresentation

18

Risk Management and Insurance Planning

Workbook

177. Which of the following statements is (are) true with respect to the human life value approach? I. The human life value approach considers all sources of income that the family receives. II. The human life value approach does not consider the time value of moneyfuture cash flows are not discounted back to present value. a) b) c) d) II only Neither I nor II I only Both I and II

183. Which of the following statements is (are) true with respect to Workers Compensation and Employers Liability Insurance? I. Workers compensation insurance is characterized by deductibles and coinsurance. II. Workers compensation provides coverage for occupational injury and disease. a) b) c) d) I only II only Neither I nor II Both I and II

178. An individual who is injured as result of a tort may bring a legal action against the person who caused the injury. In a court of law, the injured party is known as the _____. a) b) c) d) Tortfeasor Plaintiff Attorney Defendant

184. An individual may commit an act that results in bodily injury or damage to someones property. A court of law may order the person responsible for the wrongful act to pay damages to the party who was injured. This type of risk is called ___________ a) b) c) d) Speculative risk. Property risk. Liability risk. Fundamental risk.

179. In some situation, it is not necessary to prove fault for injuries resulting from an act for the act to be deemed negligent. Such situation are described as involving : a) b) c) d) Comparative negligence. Contributory negligence. Breach of contract. Strict liability.

180. Conditions that increase either the frequency or severity of loss are called : a) b) c) d) Subrogation. Risks. Hazards. Perils.

181. What is the main responsibility of the underwriting department of a life insurance company? a) To guard against adverse selection. b) To set a limit on the amount of insurance issued. c) To set adequate insurance rates. d) To avoid exposures that could result in loss. 182. Liability arising out of work done by independent contractors is known as : a) b) c) d) Contingent liability. Completed operations liability. Premises liability. Contractual liability.

185. Two ways of assessing life insurance needs is a need based approach and the other is the income replacement method. What in your judgement would be the additional life cover required for Mr.Rao on the basis of each of the two approaches. Mr. Rao is the sole income earner in the family. Mrs. Rao is a homemaker. They are aged 40 and 36 respectively. Life expectancy for both of them is another 40 years. They have no children. Other information you have is: Current investment port folio - Rs. 20 Lakhs; Estimated final Expenses - Rs. 1 Lakh; Present annual expenses-Rs. 3 Lakhs (including a Lakhs of Mr. Raos personal expenses); Mr. Raos post-tax income in hand-Rs. 3.5 Lakhs; Assume a post tax, post inflation return/ discounting factor of 3%. a) b) c) d) Rs. Rs. Rs. Rs. 28 Lakhs; 46 Lakhs; 26 Lakhs; 28 Lakhs; Rs. 40 Lakhs Rs. 58 Lakhs Rs. 58 Lakhs Rs. 60 Lakhs

Workbook

Risk Management and Insurance Planning

19

Use the information below to provide answers to Questions 186 to 188 Your client earns Rs. 60,000 annually. She is married and has a child aged three. If she dies, she wants her family to receive Rs. 45,000 annually for the next 20 years, with the first payment due on her death, and to provide Rs. 120,000 for her childs college education in fifteen years time. She also wants to set up a final expense fund of Rs. 15,000, and pay off the mortgage, automobile loan, and outstanding credit card balances. Extracts of her personal balance sheet are as follows: Assets Condominium Personal Property Automobile Mutual Funds Bank Deposits Life Insurance Mortgage Reducing Term Assurance Group Death Benefit Total Liabilities Automobile Loan Mortgage Credit Card Balance Total

188. The minimum amount of additional life insurance your client should purchase to attain her financial goals. a) b) c) d) No insurance is needed 14,3000 152,000 440,000

189. Calculate half yearly premium on SA 150000/-on the basis of the following data: Plan Term Endowment 25 years, tabular premium Rs. 53.40 per thousand Accepted with DAB extra Re.1 per thousand S.A. Occupation Extra Rs. 3 per thousand S.A. Rs. 1, 50,000 Rebate Rs. 2 Yearly Mode = Rebate less 3% Half yearly premium Rebate Less 1.5% Quarterly premium Rebate Nil Monthly premium Rebate plus 5% a) b) c) d) 8070 4095 7740 8610

Rs. Rs. Rs. Rs. Rs. Rs.

210,000 50,000 40,000 50,000 10,000 100,000

Rs. 100,000 Rs. 30,000 Rs. 590,000 Rs. 15,000 Rs. 100,000 Rs. 5,000 Rs. 120,000

190. Calculate loan amount:

Assuming that the life insurance proceeds and liquid funds can earn 6% annually, calculate 186. The present value of the annual incomes she would like to ensure for the family a) b) c) d) 500,246 516,146 547,115 900,000 Vested bonus declared from 3/88 to 3/ 91: Rs. 150 per thousand sum assured Interim bonus declared on 31.03.92: Rs. 75 per thousand sum assured. Loan: 90% of surrender value if the policy is at force or 85% of surrender value if policy is in paid up condition. 18,828 16,004 24,711 21004

187. The present value of her childs college education cost a) b) c) d) 50,071 55,921 119,827 130,875

a) b) c) d)

20

Risk Management and Insurance Planning

Workbook

191. Calculate paid-up value: a) b) c) d) Sum Assured Rs. 1, 00,000 Plan Without Profit Endowment Term 20 years Date of commencement 20.02.1995 Last premium paid 20.02.2000 Quarterly Mode Date of Birth 20.02.1950 Tabular premium 50.05 25,000 50,000 26,250 30,250

194.

Which of the following is needed to calculate the clients human life value? i. Average annual earnings to the age of retirement.

ii. Selection of an appropriate capitalization rate iii. Costs of self-maintenance. iv. Number of years from the clients present age to the contemplated age of retirement. a) b) c) d) i, ii i, ii, iii i, iii, iv i,ii, iii,iv

Use the information provided below to provide answers to Questions 192 and 193 The sales literature of an insurer, pertaining to a particular participating policy, claims that in the past, the company had paid maturity benefits of two-and-a half times the basic sum assured on their 15-year endowment policies. The company uses the uniform compound reversionary bonus system and the terminal bonus system to distribute profits to its participating policyholders. Assuming a uniform reversionary bonus rate of 35 per thousand sum assured was declared over the last 15 years, a basic sum assured of 50,000 and level annual premiums of 5,066. 192. Ignoring mortality, calculate the terminal bonus content in the maturity proceeds a) b) c) d) 50,000 48,750 83,767 75,000

195. When fine arts or antiques are insured under a homeowners policy by an endorsement, a) Coverage is usually on a replacement cost basis. b) Coverage is usually on an actual cash value basis. c) Coverage is usually provided on a valued basis. d) The perils are the same as the homeowners policy to which the endorsement is attached. e) Coverage limits are the same as the homeowners policy to which the endorsement is attached. 196. Which of the following is / are true regarding the ownership of life insurance? 1. A policy can only be issued to the insured. 2. Generally, assigning a policy requires proof that the insured is still insurable meaning still in good health. 3. Only a person with an insurable interest, generally a relative, a business associate, or lender, can be named as a beneficiary. 4. The owner can assign (transfer) the policy to whomever he or she chooses, even if the assignee has no insurable interest. a) b) c) d) 1, 2, and 3 only. 1 only. 2 and 4 only. 4 only.

193. Ignoring mortality, calculate the internal rate of return implicit in the maturity proceeds. a) b) c) d) 5% 10% 7% 8%

Workbook

Risk Management and Insurance Planning

21

197. The following are statements made concerning contracts of insurance. Identify the statement/s that is/are correct. I. Property and liability insurance contracts are freely assignable by Policy owners without the insurers prior approval. II. The requirement that the insured must cooperate with the insurer in defending a liability claim is an example of a condition precedent. a) b) c) d) I only II only I and II Neither I nor II

202. Maximum sum assured available under a Life insurance policy currently is Rs. ______ Lakhs a) b) c) d) 50 75 100 No ceiling is mandated

203. TPAs are being employed currently in _______ policies. a) b) c) d) Medical Insurance Motor Insurance Life Both (a) and (b)

198. A son cannot insure his parents life. a) True b) False c) True only if both parents are earning a regular income. d) True only if the son is dependent on the parents. 199. _________ insurance policy cannot be assigned. a) b) c) d) Term Endowment Money back ULIP

204. Mrs. Mishra is a female assessee aged 55. The maximum amount of income tax deduction that she will get on account of payment of premium on a Medical Insurance policy is Rs. _____ a) b) c) d) 10000 12500 15000 20000

205. The Insurance Act was passed in _______ a) b) c) d) 1956 1938 1872 1881

200. An assigned life insurance policy can revert to the original policy holder once the liability is paid off. a) True b) False c) It can revert only at the discretion of the assignee. d) True, but the original policy holder will not receive any of the interim bonuses which have been declared on the policy. 201. The minimum capital required for an insurance company as per the IRDA guidelines is Rs. ______ crores. a) b) c) d) 50 75 100 200

206. Clause pertaining to Contribution and Subrogation are contained in ______ insurance contracts. a) b) c) d) Life Medical Accident Pension Plan

207. The cost of insurance premium is ________ proportional to the odds of an event occurring. a) b) c) d) Directly Inversely Indirectly Data insufficient

208. Wearing a helmet while riding a bike is an example of _______ a) b) c) d) Risk Retention Risk Control Risk Sharing A combination of all the three options

22

Risk Management and Insurance Planning

Workbook

209. A start-up coming approaching the stock market with an IPO is an example of ______ a) b) c) d) Risk Retention Risk Reduction Risk Sharing A combination of all the three options

215. No claim bonus is not available in a _______ policy a) b) c) d) Motor Insurance Health Insurance Home and Content Both (a) and (b)

210. Mr. Vinod has purchased medical insurance policies from two companies viz. X and Y for a sum of Rs. 5 Lakhs each. He files for a claim of Rs. 5 Lakhs from X, who reimburses him for the same. X then approaches Y and recovers half this amount from it. X has employed the principle of _______ a) b) c) d) Contribution. Subrogation Collaboration Intimidation

216. Income, Income plus expenses, Multiple of salary are all methods of calculating insurance need by __________ a) b) c) d) Research Rule of thumb Past record None of the above

211. The following are examples of _______. I. Outstanding balance on credit card II. Short-term loans outstanding III. Bank overdraft a) b) c) d) Long term liabilities Current liabilities Long term assets Current Assets

217. A client explains that she only wants an insurance policy that will cover her family against financial risk over the next five years, while she still has dependent children and a large mortgage. It is unlikely her income will increase over this period. What type of insurance is she looking for? a) b) c) d) An unit linked insurance plan Money back policy Term insurance with a level premium Term insurance with a stepped premium

212. In India, life insurance has been traditionally sold more as a __________ product. a) b) c) d) Protection Investment Tax-saving None of the above

218. Which of the following is type of pecuniary insurance? a) b) c) d) Commercial vehicle insurance Engineering insurance Money policy Workmans compensation

213. The rights of a beneficiary under an MWP life insurance policy are not affected by divorce. a) b) c) d) True False False in the case of Hindus True only in the case of Muslims

219. Participating policies are those where ____. a) Both insurer and insured participates in each others loss b) Insured participates in running of insurance of company c) Insured participates in surplus of insurance company d) Insurer participates in loss of insured 220. There is an excess of Rs. 5000 for a health insurance policy. In a case where there is an expense of Rs. 50000 for the insured then the insurance company will pay ______ a) b) c) d) Rs. Rs. Rs. Rs. 5000 45000 50000 75000

214. A general insurance will usually insist that ________ is professionally valued before providing a Homeowners Insurance Cover. a) b) c) d) Residence Electronic items Jewellery Furniture

Workbook

Risk Management and Insurance Planning

23

221. Anand was driving his car home from work, when a pit dug by the municipal corporation in the road, remained open and unmarked. He met with an accident and had to be hospitalised for 3 months. What are the insurance claims that can a rise from this accident? a) Anand can claim personal insurance for the accident, as it was not caused by negligence on his part; the municipal corporation cannot claim third party loss insurance to pay damages to Anand, as it was negligent. Anand can claim insurance for damage due. b) Anand can claim temporary disability insurance and insurance for his damaged car c) Anand has to apply to the municipal corporation for damages, which the corporation will pay out of its claims for liability to third party. His motor insurance will cover damages to his car. d) Since the municipal corporation was negligent, it would not be able to lodge a claim to recover payment of damages to Anand. Anand will only receive motor insurance claims on his car. 222. Sujata was standing on the terrace of her building hanging out clothes. She accidentally fell off and landed on the sunshield of the next floor, which crashed and damaged the car of her neighbour parked below. What are the insurance claims that arise from this event? a) Sujata can claim personal accident insurance. Both her neighbours will claim property insurance for the freak accident. b) Sujata cannot claim accident insurance as the accident was caused by her negligence. Her neighbours can claim property insurance cover for loss to their property c) Sujatas neighbours will collect damages from her, which Sujata can pay out of insurance cover for losses to third parties. d) Sujatas neighbours will not be able to claim insurance, as the damage to their property due to such freak accidents is not usually covered by insurance. Sujata will be able to claim her accident insurance, as she did not fall intentionally.

223. During retirement, a person would maintain a high level of life insurance protection only to: 1) Pay estate duty 2) Provide succession or dependent income 3) Repay outstanding debt 4) Build cash value a) b) c) d) (1) and (2) (1) and (3) (1), (2) and (3) None of the above

224. Your client has bought life insurance and medical insurance, but has not bought a cover for permanent disability. His argument is that he is paying too much by way of premium for risks that he believes are far-fetched and not likely to affect him. What would you advise the client? a) A financial planner can guide the client to consider the losses from permanent disability and highlight the risks to the client and recommend an appropriate policy for him. b) If a client is not willing to bear the costs of premium, it can be assumed that he is willing to bear the costs of risk retention. Insurance may not be necessary in such cases. c) If losses that would occur to the client in the event of permanent disability are higher than what he can bear, the client is better off buying insurance. The costs of insuring against losses, which have lower probability of happening, will in any d) The amount of insurance a person will buy depends on his perception of risks and their impact on him. It would not be possible to persuade this client to buy more insurance.

24

Risk Management and Insurance Planning

Workbook

225. Suresh has not bought accident insurance cover, though his two-wheeler is covered for damages from accidents. He wears a helmet and drives carefully. What can you say about his risk management? a) Suresh has insured the property risk. He controls some of his personal risk and retains the rest of the risk. b) Suresh has controlled his personal risk and insured his property risk c) Suresh has not done anything to manage his risks and has to immediately go for accident and personal risk cover. He cannot rely on third party damages alone to cover the risk of the road. d) Suresh has transferred his personal risk to other drivers of the road, insured his property risk and can claim damages if accidents are caused by third party negligence. 226. Mrs. Rangnekar, a 40-year-old widow, has a 8-year-old son. Her current savings are not adequate to provide for her sons post graduate studies however she will be able to save it up by the time he finishes graduation i.e. when he is 20 years old. Mortality tables indicate that her life expectancy is another 30 years. Which of the following is true? a) b) c) d) She needs to insure her life for 12 years She does not need to insure her life She needs to insure her life for 30 years She needs to insure her sons life for 30 years

228. The gross income of a person is at Rs 3 Lakhs when the average tax rate is 20%. Due to the receipt of an additional sum of Rs. 2 Lakhs the average rate goes up to 25%. If there is then an additional sum of Rs. 50000 spent for own expenses then the amount remaining for t he f amily is _________. a) b) c) d) Rs. Rs. Rs. Rs. 325000 375000 425000 450000

229. Personal Data of Nilesh: Earnings: Rs. 2 Lakhs p.a Debts: Rs. 1,00,000 Total Expenses: Rs. 1,28,572 To support dependents 75% of predeath salary (if no debt)

To support dependants spouse and one child, 75% of pre-death salary required (if no debt) Assuming simple interest of 7% p.a, As a Financial Planner find out the Insurance required by Capital Retention Approach and the times it will cover his current annual salary? a) Rs. 22,42,000 and 10 times of current annual salary b) Rs. 22,42,000 and 12 times of current annual salary c) Rs. 21,42,000 and 11 times of current annual salary d) Rs. 21,42,000 and 12 times of current annual salary e) Rs. 8,65,300 and 5 times of current annual salary f) None of the above 230. Brijesh has a health insurance policy with sum assured Rs. 10 Lakhs. The deductible is Rs.2,500. Brijesh had a operation to remove his appendix which cost him Rs. 30,750. He also lost Rs. 5,000 in wages as he could not work for 2 weeks. What would be the amount paid by the insurance company? a) b) c) d) Rs. Rs. Rs. Rs. 30,750 28,250 33,250 35,750

227. Holistic Institute of Wealth Management has a great Education Centre and a f ull business. Dr.Shah, Head Academics is held in high esteem nationwide for his expertise, knowledge and lecture delivery. He has made a outstanding contribution for the last 8 years to enable market leadership for Holistic Institute of Wealth Management. The most appropriate policy that the owners may take in the interests of business on the life of Dr. Shah would be __________. a) Personal Accidental Insurance b) Term Insurance c) Key Man Insurance

Workbook

Risk Management and Insurance Planning

25

231. Normal medical policies do not have a loss of earning cover. Which one of the following does not come under principal of merit rating? a) b) c) d) Schedule rating Experience rating Retrospective rating Judgement rating

In this case his immediate cash needs are ________ a) b) c) d) Rs. Rs. Rs. Rs. 170000 370000 220000 350000

232. Bima Kavach Yojana is a policy designed for area a) b) c) d) Urban Area Rural Area Both Urban & Rural Area None of them

238. If the Readjustment Period Expenses at todays value is Rs. 3 Lakhs and the Dependency Period Expenses at todays value is Rs. 12 Lakhs, plus the Blackout Period Expenses are Rs. 5 Lakhs, As a Financial Planner find out his approximate Net Income Needs. a) b) c) d) e) Rs. 25.70 Lakhs Rs. 15 Lakhs Rs. 20 Lakhs Rs.. 23.7 Lakhs None of the above

233. Which one of the following is not the Money Back Policy of LIC? a) b) c) d) Jeevan Chhaya Jeevan Anand Jeevan Surbhi Jeevan Rekha

234. Which professionals are not eligible for Professional Indemnity Cover? a) b) c) d) Plastic Surgeons General Physicians Dentists Consulting Physicians

239. In the above case, as a Financial Planner find out his Insurance Requirement as per Needs Approach. Note : His Investments in Mutual Funds and Stocks are valued currently at Rs. 8 Lakhs. He has an exisiting Insurance Policy with a Sum Assured of Rs. 2.70 Lakhs. a) b) c) d) e) Rs. 25.70 Lakhs Rs. 17.70 Lakhs Rs. 15 Lakhs Rs. 11 Lakhs None of the above

235. Rajarajeswari Mahila Kalyan is a policy designed for women, it comes under which category of Insurance? a) b) c) d) Health Plan Money Back Term Plan Personal Accident

236. In Mediclaim, what is the limit of premium which is totally exempt from Income Tax? a) Rs. 10,000, if both the partners are covered b) Rs. 10,000, if paid by a cheque c) Rs. 15,000, for family package d) Rs. 20,000, if insurer has life insurance policy 237. A person is looking at evaluating the life insurance needs that will arise for him. The estimate is that final medical expenses will be around Rs. 1.5 Lakhs, the funeral charges Rs. 20000. In addition his wife will require Rs. 2 Lakhs in her retirement period.

240. A person lives in a nuclear family consisting of his wife and two children. He wants to assume his premium needs at 5 per cent for himself plus 1 percent for other members of his family. He has an income of Rs. 5 Lakhs and an expense figure of Rs. 3 Lakhs during the year. The expense is expected to go upto Rs. 3.25 Lakhs next year. The amount of insurance cover required here is ______. a) b) c) d) Rs. 5 Lakhs Rs. 10 Lakhs Rs. 15 Lakhs Cannot be determined

26

Risk Management and Insurance Planning

Workbook

241. A person lives in a nuclear family consisting of his wife and two children. He wants to assume his premium needs at 5 per cent for himself plus 1 percent for other members of his family. He has an income of Rs. 5 Lakhs and an expense figure of Rs. 3 Lakhs during the year. The expense is expected to go upto Rs. 3.25 Lakhs next year. The maximum amount of premium that will be used for buying insurance cover will be ________ a) b) c) d) Rs. 40000 Rs. 45000 Rs. 50000 Cannot be determined

244. In case of a need that will amount to Rs. 3 Lakhs for 5 years after a period of 3 years and where the rate prevailing is 5 % then this figure converted to an insurance need today comes to _________. a) b) c) d) Rs. Rs. Rs. Rs. 1178089 1121989 1356474 1456786

245. Which of the following statements is / are true with respect to pure risks? i. Pure risks may produce either a profit or a loss. ii. Premature death or damage to property caused by a fire are pure risks. a) b) c) d) i only Both i and ii Neither ii nor i ii only

242. A person lives in a nuclear family consisting of his wife and two children. He wants to assume his premium needs at 5 per cent for himself plus 1 percent for other members of his family. He has an income of Rs. 5 Lakhs and an expense figure of Rs. 3 Lakhs during the year. The expense is expected to go upto Rs. 3.25 Lakhs next year. As a Financial Planner find out the amount of premium that will be used for the insurance cover by employing Premium as a Percentage of Income Method. a) b) c) d) e) Rs. 40000 Rs. 41000 Rs. 16000 Rs. 14000 None of the above

246. Karan wants to withdraw Rs. 1200/- at the end of each month for the next 5 years. He expects to earn 10% interest compounded monthly on his investments. What lump sum should he deposit now? a) b) c) d) Rs. Rs. Rs. Rs. 56949 58630 56478 59119

247. From the following particulars, calculate paid-up value: a) b) c) d) Sum assured: Rs. 50,000, Plan: Endowment without profit Term: 20 years Date of commencement: 19/02/1994 Last premium paid yearly: 14/02/2000 Rs. Rs. Rs. Rs. 50,000 17,500 15,000 18,000

243. A person lives in a nuclear family consisting of his wife and two children. He wants to assume his premium needs at 5 per cent for himself plus 1 percent for other members of his family. He has an income of Rs. 5 Lakhs and an expense figure of Rs. 3 Lakhs during the year. The expense is expected to go upto Rs. 3.25 Lakhs next year. As a Financial Planner find out t he Total Insurance cover he can avail by using the Premium as a Percentage of Income Method (Note Insurance Premium for his age is Rs. 4.00 per Rs. 1000 sum assured). a) b) c) d) e) Rs. 62.5 Lakhs Rs. 100 Lakhs Rs. 40 Lakhs Rs. 102.50 Lakhs None of the Above

Workbook

Risk Management and Insurance Planning

27

248. Case 1: The owner of a building with running machinery seeks to insure the property by choosing a certain sum assured, as full value of the risk to be covered. Case 2: The owner of a property suffered a fire loss. He seeks to restore the cover value to its original level to make up for the loss and is willing to pay additional premium for this purpose. (A) The former is a case of reinstatement as a basis of indemnity, while the second instance is one of reinstatement of sum insured following a loss under the policy. (B) No, it is in reverse order. a) b) c) d) Statement A is right Statement B is right Both statements A & B are wrong None of the above

251. Nilesh owns a laptop that was stolen. The laptop cost Rs. 35,000 when it was purchased one year back. A similar laptop can be bought today for Rs. 20,000. Assuming that the laptop was 50% depreciated, what is the amount payable by the insurer? Assume deductible to be Rs. 1,000. a) b) c) d) Rs. 9,000 Rs. 10,000 Rs. 19,000 Rs. 34,000

Use the following data for questions 252 to 254 Current Age: 40 Sum Assumed : Rs. 1,00,000 Annual Premium : Rs. 2,280 Projected dividends : Rs. 15,624 Cash Value after 20 years : 35260 Also assume that if dividends are invested at 5% interest they will grow to Rs. 24,400 at the end of 20 years. Assume if the premiums are invested at 5% interest they will grow to Rs. 79,159 at the end of 20 years. A Re 1 deposit at the beginning of each year at 5% interest will accumulate to Rs. 34,719 at the end of 20 years. 252. The net cost of insurance per Rs. 1,000 based on the traditional net cost method will be: a) b) c) d) Rs. (-2.64) Rs. 1.499 Re. 0.517 Rs. 5.284

249. Ashish purchased a sofa set for Rs. 50,000 and insured it on an actual cash value basis. Three years later the sofa set was destroyed in a fire. At the time of the loss, the furniture had depreciated to 50% of its value. The replacement cost of a new sofa set at the time of the loss was Rs. 60,000. The deductible was Rs. 3,000. What amount would Ashish get from the insurance company? a) b) c) d) Rs. Rs. Rs. Rs. 47,000 57,000 27,000 17,836

250. There are 2000 buildings in a city valued at Rs. 20,00,000 each. All the buildings are insured upto 50% of their value. The following fire losses occur: 2 total losses 30 partial losses at Rs. 2,00,000 each.

253. The net cost of insurance per Rs. 1,000 based on the surrender cost method will be: a) b) c) d) Rs. (-0.264) Rs. (-0.404) Rs. 5.616 Rs. 1.577

What would be the pure premium rate? a) b) c) d) Rs. Rs. Rs. Rs. 1.25 per 2.00 per 2.50 per 4.00 per thousand thousand thousand thousand

254. The net cost of insurance per Rs. 1,000 based on the net payment cost method will be: a) b) c) d) Rs. (-0.264) Rs. 1.499 Rs. 1.823 Rs. 15.77

28

Risk Management and Insurance Planning

Workbook

Use the following data for questions 255 to 257 Current Age: 25 Sum Assumed : Rs. 20,00,000 Annual Premium : Rs. 24,860 Projected dividends : Rs. 81,400 Cash Value after 20 years : 4,31,400 Also assume that If premiums are invested at 5% interest, they will grow to Rs. 8,63,100 at the end of 20 years. If the dividends are invested at 5% interest, they will grow to Rs. 1,16,300 at the end of 20 years. A Re. 1 deposit at the beginning of each year at 5% interest will accumulate to Rs. 34,719 at the end of 20 years. 255. The net cost of insurance per Rs. 1,000 based on the traditional net cost method will be: a) b) c) d) Rs. Rs. Rs. Rs. (-0.870) (-0.780) 20.790 2.1745

What would be the amount paid by the insurance company? a) b) c) d) Rs. Rs. Rs. Rs. 20,000 10,000 17,000 12,000

259. Ashok, born in 1950, has a life expectancy at birth of 65 years. Sita, his wife, born in 1955 has a life expectancy at birth of 70 years. Assuming that the life expectancies have not changed, Ashok is planning to buy an annuity in 2008 to be paid to him or his wife till anyone of them is alive. He should buy an annuity for ________ years. a) b) c) d) 10 yrs. 12 yrs 07 yrs 17 yrs

260. For dependent parents, the least important risk to be covered is a) b) c) d) Life insurance cover Hospitalization / Sickness Insurance Disability Insurance Long term Care cover

256. The net cost of insurance per Rs. 1,000 based on the surrender cost method will be: a) b) c) d) Rs. (-0.780) Rs. 9.084 Rs. 21.510 Rs. 4.542

257. The net cost of insurance per Rs. 1,000 based on the net payment cost method will be: a) b) c) d) Rs. (-0.780) Rs. 9.084 Rs. 21.510 Rs. 10.75

261. Suresh, the driver, driving Rams car (Ram was traveling in the car) was involved in a collision accident against the boundary wall of Rahuls house and also injured Rahul, Rams neighbour. It was later ascertained that driver Suresh was under the influence of alcohol. One of the perils mentioned below will not be covered while setting the claim: a) Repair charges to Rams car b) Boundary wall repair charges of Rams house c) Medical treatment charges of Ram d) Medical treatment charges of Rahul 262. For guaranteed renewability of term insurance policies, which of the following are important factors? a) b) c) d) Age of the insured Premium rates Risk status All of the above

258. Ramesh has a family medical insurance policy with sum assured of Rs. 1,00,000. The policy has a deductible of Rs. 2,000 that applies to each covered person. He and his family are injured in an accident and have to be hospitalized for treatment. The treatment expenses of each are: Ramesh: Rs. 2,500 His Wife: Rs. 10,000 His Daughter: Rs. 7,000 His Son: Rs. 500

Workbook

Risk Management and Insurance Planning

29

263. A father takes out a policy to meet his sons enhanced educational needs when the son reaches 18 years of age. Unfortunately after paying 5 years premium, the father dies. By what means can the policy be kept in force without payment of further premiums so that the intended benefits are retained? a) b) c) d) Accident benefit premium Lien on policy Premium waiver facility None of the above

269. Premium is the legal consideration paid by the ____________ a) b) c) d) Insurer Insured Agent Broker

270. Rate of premium depends on the basis of ________ a) b) c) d) Age Term Policy type All of the above

264. For professionals with limited time span of income earning possibilities, Which of the following plan of insurance would be ideal? a) b) c) d) Whole life Term assurance Immediate annuity Limited payment endowment plan

271. The amount required to meet the risk of death for a given age in a particular year is called ________ a) b) c) d) Risk premium Net premium Gross premium None of the above

265. Under an Overseas Travel Insurance policy, the maximum cover for third party liability is a) b) c) d) US $ 1,00,000 US $ 2,00,000 US $ 5,00,000 None of the above

272. The proposal form includes : a) b) c) d) Personal details of the client Policy details Information about the insure Risk factors in insurance business

266. Which of the following statement(s) is / are True? Statement (A): The existence of insurance may make people to exert less effort to control losses. Statement (B): It might also induce commitment of frauds against insured. a) b) c) d) Both A & B above are true Statement A is true Statement B is true Both A & B are not true

273. The proposal form has to be signed by : a) The proposer and witnessed by a third party b) The proposer and the agent c) The proposer and the person whose life is to be assured d) Both A and C 274. The surplus generated from the investment division of an insurance company, which is distributed amongst the policyholders is called as ___________ a) b) c) d) Bonus Interest Maturity value None of the above

267. A Term Insurance plan can be for _______ a) b) c) d) Constant cover Decreasing cover Increasing cove All of the above

268. The actuary is responsible for _________ a) b) c) d) Product design Premium rates fixation Paid up and surrender value fixation All of the above

275. Every policyholder is given the option to choose premium payment ___________ a. Mode b. Frequency c. Both A and B

30

Risk Management and Insurance Planning

Workbook

276. Term insurance provides _______ benefit a) b) c) d) Death Survival No Maturity

281. Which of these is involved in Needs Analysis? a) b) c) d) Needs Explaining Needs Sequencing Needs Prioritization None of the Above

277. An owner of a large group of privately owned residential property units is seeking cover from a general insurance company for (i) fire and other associated perils (ii) depreciation in the value of property (iii) complete destruction of property from terrorist strikes for the next six months. The chances are that the insurance company will ____ a) b) c) d) Offer cover for (i), (ii) and (iii) Offer cover for (i) and (iii) Offer cover for (i) only Offer cover for (i) and (ii)

282. The prescribed training program for life insurance agency that an applicant has to undergo is for ________. a) b) c) d) 50 hours 150 hours 80 hours 100 hours

283. A Broker is characterized by the fact that he is: a) Attached to the insurance company b) Representative of a group of companies c) An independent intermediary d) Employed by the insurance company 284. Which agency conducts the pre-recruitment test for a prospective agent? a) b) c) d) The Insurance Company Insurance Institute of India IRDA Actuarial Society of India

278. Which of these are not contracts of indemnity? a) b) c) d) Life insurance Fire insurance Homeowners insurance Baggage insurance

279. Consider the two statements below: (A) While assessing financial liabilities, it is customary to include credit card debt, hire purchase of household goods, lease etc. under short term liabilities. (B) Longer term liabilities include personal loans etc. a) b) c) d) Statement A alone is correct Statement B alone is correct Statements A & B are correct None of the above

285. The IRDA allows a cooling period of ______ for the holder of a Life Insurance policy. a) b) c) d) 10 days 15 days 30 days 07 days

286. A Composite Broker may be defined asa) A general and life insurance broker b) One who carries on business as a broker and agent c) An insurance and reinsurance broker d) A general insurance agent 287. Within how many days of accepting a proposal must an insurer handover the copy to the insured? a) b) c) d) 10 days 15 days 07 days 30 days

280. Consider the two statements below: (A) Past losses may be a precursor of further losses in all types of life and general insurance business. (B) Past losses may be a precursor of future losses in some general insurance cases, critical illness and personal accident insurance cases. a) b) c) d) Statement A alone is correct Statement B alone is correct Statements A & B are correct Statements A & B are incorrect

Workbook

Risk Management and Insurance Planning

31

288. Consider the two statements below: (A) An important step in risk control is to spread out the cost of risks evenly over a period of time. (B) The objective should be to minimize the cost of risk. Of the above two statements, what should be the correct order of objectives a) Both A & B are of equal importance b) The given order: first A then B is correct c) B should precede A d) None of the above 289. Life expectancy has been on the rise in India since 1921 because ofa) b) c) d) An improvement in dietary practices Safer birth practices Enhanced health care amenities All of the above

294. Which of these is provided for under a Term Insurance policy? a) Only the premium is paid till the date of death b) Sum assured in case of death c) Sum assured plus bonus in case of death d) None of these 295. A Participant Whole Life policy will allow the insurer to participate ina) The day to day running of the insurance company b) The board of the insurance company c) The profits of the company d) None of these 296. Mr. Singh has applied for a bank loan of Rs. 1 crore to launch his own business venture. He is an Electronics Engineer by profession. The bank approves his loan with a conditional hypothecation of +25% and requests him to hypothecate the life policy as collateral security for mortgage redemption. Calculate the sum assured of the policy. a) b) c) d) 80 Lakhs 75 Lakhs 1.25 crore 2 crores

290. Which of these will NOT be considered a dependant while taking insurance? a) b) c) d) Children Relatives A spouse or partner None of these

291. One must look at various stages of an individuals life to analyze his/her insurance need. These stages of life comprise a) b) c) d) Young family Young adult Matured couple All of these

297. __________ is a claim arising out of the life assured, surviving till the end of the selected term of the life insurance: a) b) c) d) Annuity Maturity claim Death claim None of the above

292. Which of these insurances are appropriate for a young adult? a) b) c) d) Household insurance Personal protection insurance Automobile insurance All of these

298. An Endowment policy may be defined as one that pays: a) b) c) d) Premiums on death only Premiums paid on death or survival Assured benefits on death only Assured benefits on death or survival

293. Liability exposure is NOT a) b) c) d) Self - injury in an accident Injury to employees Bodily injury or property damage Professional negligence in giving advice

299. The disadvantage of insurance mechanism includes a) b) c) d) Reduction of uncertainty Operating expenses and moral hazard Capital gain and financial risk None of the above

32

Risk Management and Insurance Planning

Workbook

300. Which of these policies refer to a Home Loan/ Mortgage Protection? a) b) c) d) Credit insurance policy Property insurance policy Policy guaranteeing the mortgage loan None of the above

306. The insurance provided by critical illness covers a) b) c) d) Assured benefits for major illnesses Hospital expenses for major illnesses Long-term care after a major illness All of these

301. Which of these is NOT provided for by an Educational Life policy? a) Continued Educational support to an insured individuals children, in case of his death b) Education to insured individuals up to a certain age c) Payment of educational expenses at a future time d) All of these 302. What is a Cooling off provision in an insurance policy? a) When the insured individual can elect to cancel the policy and receive refund of the premium b) When the insured individual does not have to pay a premium c) When the insurance company cools off or stops its operations d) None of these 303. If a claim arises out of the death of an insured individual, _____ would be paid the money. a) b) c) d) the first son of the insured individual the spouse of the insured individual the mother of the insured individual the person nominated under the policy by the individual, if any

307. Income Protection Insurance allows for protection against loss of income in which of these instances? a) A business loss b) The insured individuals disability or sickness c) Unemployment of the insured individual d) Death of the insured individual 308. A Hospital Expenses Insurance policy will NOT cover: a) Expenses incurred due to outstation treatment b) Pre and post hospitalization expenses c) Hospitalization expenses following an accident injury d) None of these 309. When there is the possibility of loss or no loss in an outcome, it is a case of ______ a) b) c) d) Gambling Speculative risk Pure risk Special risk

310. During the review process of the insurance plan, the planner must __________ a) educate the client about the need for additional cover in case of changed circumstances b) let the client know if any new product will be more suitable for the client in place of the existing c) notify the client of the risks faced by him, which are not covered by his program d) all of these 311. The _____ initiates the action with respect to maturity claims under a life insurance policy. a) b) c) d) planner policy holder intermediary who sold the policy insurance company

304. Insurance under Disability Insurance is not provided for___________ a) Disabilities caused by sickness b) Disabilities caused by accidents c) Disabilities caused by hereditary causes d) None of these 305. Under personal accident insurance, the amount insured is associated with _____ a) b) c) d) Income of the insured individual Family size of the insured individual Age of the insured individual Personal habits of the insured

Workbook

Risk Management and Insurance Planning

33

312. The claim will be payable to _____ if the policy owner and the insured individual are different. a) the policy owner b) the spouse and children of the insured individual c) the legal heirs of the life assured d) the person nominated to receive the policy money 313. Under a disability benefit policy, the waiting period is the period of time that must elapse ______ a) before the issue of the policy document b) before the policy benefit will commence c) before the proposal can be considered d) before the settlement of claim after submission 314. When should one inform the insurer in case of a health insurance claim? a) When the insured person is hospitalized b) When the health disorder is noticed c) When the course of treatment for the insured person is complete d) When the bills of the hospital, where the insured individual was hospitalized, are paid. 315. Which of these is applicable under property insurance? a) The amount payable is at the sole discretion of the employer b) The amount payable is guaranteed under the policy c) The amount payable is assessed according to the principle of indemnity d) None of these 316. Which of these claims is applicable under disability benefit claims? a) The policy holder has to undergo a medical examination by an approved medical examiner b) The policy holder has to submit medical certificates from the attending doctor c) The policy holder has to undergo specific medical examinations d) All of these

317. The insurance planner or intermediary is responsible for_________. a) Health insurance claims b) Death claims in life insurance policies c) Disability claims in personal accident policies d) All of these 318. _______ is the regulator for the insurance industry in India a) Controller of Insurance b) Insurance Regulatory and Development Authority c) General Insurance and Reinsurance Business d) Tariff advisory committee 319. Consider the two statements below: i. In life insurance, where the proposer was born would be deemed a material fact. ii. In personal accident insurance, medical history would be deemed a material fact. a) Only statement i is wrong. b) Both statements i & ii are correct c) Statement i is correct, while ii is wrong. d) Both Statements I & II are incorrect. 320. An Indian Insurance Company can perform the following: a) Life insurance and Reinsurance business b) Life and General insurance business c) General Insurance and reinsurance business d) Either life or general insurance or reinsurance business 321. General insurance business comprises _______ a) b) c) d) Fire insurance Marine insurance Miscellaneous insurance business All of these

34

Risk Management and Insurance Planning

Workbook

322. _______ transacted general Insurance business before the IRDA act was passed in 1999 a) General Insurance Corporation of India (G.I.C) b) General Insurance Corporation of India along with its 4 subsidiaries c) Life Insurance Corporation (L.I.C) d) All of these 323. What is the minimum paid up capital required for establishing a new Life Insurance Company? a) b) c) d) Rs. 200 crore Rs. 100 crore Rs. 50 crore As fixed by the IRDA

326. Ram and Raja are healthy, able bodied men. Ram lives on a flood plain and Raja lives on a hill next town. While talking out insurance, t he insurance company proposes a higher premium for Ram. How would you discuss the present situation? a) Both are healthy, so both should be charged the same premiums b) They should get a discount for being healthy c) Living on a flood plain does not in any way affect the premium d) Ram should understand that that the risk is higher so the premium is also higher 327. A property worth Rs. 10 Lakhs has been preferred to have a cover of Rs. 6 Lakhs by its owner. Claim has been logged for a loss of Rs. 2 Lakhs. How much does the insurance company usually pay? a) b) c) d) Rs. 2 Lakhs Rs. 6 Lakhs Rs. 1.2 Lakhs Rs. 1.5 Lakhs

324. Consider the two statements below: i. Insurance companies reduce risk based on law of large numbers. ii. The increasing pool of resources accessible to an insurer reduces the risk of non payments. a) b) c) d) Statement ii alone is correct Both Statements i & ii are correct Both Statements i & ii are incorrect Statement i alone is correct

325. Mr. Mathew is the owner of a large group of privately owned residential property units seeking cover from a general insurance company for the following: 1. Fire and other associated perils 2. Depreciation in the value of property 3. Complete destruction of property from terrorist strikes for the next six months The chances are that the insurance company will _______ a) b) c) d) Offer cover for 1, 2, and 3 Offer cover for 1 and 3 only Offer cover for 1 only Offer cover for 1 and 2 only.

328. Mr. Mani opted to have statutory minimum cover on his car to avoid higher premiums. While saving a child, his car hit a parked car causing damage to it of Rs. 5000 and then hit the wall causing further damage of Rs. 5000; his own car suffered damages worth Rs. 8000. How much claim is admissible? a) b) c) d) Rs. 18,000 Rs. 10,000 Rs. 5,000 None of the above

329. Consider the two statements below: i II Statements made by the proposer in the proposal form are representations. All such statements are, by definition, material facts.

a. Both statements are correct b. Statement ii is incorrect c. Both statements i & ii above are incorrect d. Statement ii is correct

Workbook

Risk Management and Insurance Planning

35

330. The maximum rate of commission payable to Agents/ Advisors by the insurance companies for the first, second and the third year are ______. a. b. c. d. 25%, 10% and 7% 15%, 15% and 10% 20%, 15% and 12.5% 35%, 7.5% and 5%

refund of premiums paid. He is entitled to ______. a) b) c) d) Any refund Half the amount 75 % of the amount No refund

331. The sum assured received under any policy is taxable if the premiums payable a) b) c) d) is more than 20% of the sum assured If it exceeds 30% If it exceeds 15% If it exceeds 12.5%

335. Identify the element mentioned below that is not a factor for premium consideration in Overseas Medical/ Travel insurance. a) b) c) d) Age of the proposer Purpose of the trip Duration of the trip Amount of life insurance cover held

332. Rearrange the sequence of the planners advises to the client for resolution of insurance complaints. 1. Arbitration 2. Ombudsmen 3. In house complaint handling call of the insurer 4. Consumer Protection forums a) b) c) d) 1,2,3,4 1,3,2,4 1,4,2,3 3,2,1,4

336. Tony buys an annuity from an insurance company, under which the insurer will pay him a certain sum periodically as long as he is alive. Billy buys an annuity from an insurance company under which the insurer will pay annuities to him (or his heirs) for a definite number of years, whether he is alive or not. 1. Tony has purchased a life annuity and Billy has purchased an annuity certain. 2. Tony has annuity certain and Billy has variable annuity. a) b) c) d) 1 is correct 2 is correct Both 1 and 2 are correct None is correct

333. Following are the details of Mr. Alok. Earnings Rs. 1 Lakh per annum Debts Rs. 1,28,572 to support dependents 75% of predeath salary (if no debt). Assuming simple interest of 7% per annum, how much per current annual salary he needs for life Insurance, if multiple method is used? a) b) c) d) 10 times 8 times 12 times 16 times

337. Manish and Sheela, a retired couple, opted for a joint life survivor annuity. Under this annuity; a) The annuity is payable as long as both of them are alive b) The annuity will cease on the death of the eldest of the annuities c) The annuity is payable as long as either one of them is alive d) The annuity payment will cease on the death of any one of the annuitants 338. The least important risk, for dependent parents to be covered is _______ a) b) c) d) Life Insurance cover Hospitalization/ Sickness Insurance Disability Insurance Long Term Care Cover

334. Mr. Sippy submitted a proposal on his own life on 1.3.2003. It was completed by the insurer and the Policy Bond was received by him on 8.3.2003. On 20.3.2003, he returned the Policy Bond stating the reason for his refusal to accept some of the terms printed in the Policy Bond and asked for

36

Risk Management and Insurance Planning

Workbook

339. Consider the two statements below: 1. Type of construction is considered for Fire Insurance 2. Driving track record is a material fact for Motor Insurance a) b) c) d) Both statement 1 and 2 are correct Only statement 1 is incorrect Only statement 2 is incorrect Both statement 1 and 2 are incorrect

(2) The claims in the first case will be settled for the full value at risk while the second case will be settled on indemnity basis. a) Statement 1 is correct b) Statement 2 is correct c) Both cases will be settled on indemnity value basis d) None of the above 342. Calculate the yearly premium for the data given below: Date of Birth: 19/09/1979 Date of Commencement of Cover: 21/ 02/2005 Plan Term: Endowment with profits 25 years Sum Assured: Rs. 1,00,000 Mode Rebate: Yearly-3%, Half-yearly1.5% Sum Assured Rebate: Upto Rs.49,999 - Re. 1 per thousand Rs. 50,000 and above - Rs. 2 per thousand Tabular premium: Age 24 - Rs. 49.50 Age 25 - Rs. 51.75 Age 26 - Rs. 52.50 a) b) c) d) Rs. 4,820 Rs. 4,998 Rs. 5,020 Rs. 5,010

340. Consider the following two cases: Case 1 : After two years of taking out a life insurance policy, the assured dies after complications from emergency surgery after an acute attack of duodenal ulcer. Enquiries reveal that he had periodic stomach aches in childhood. Claimant argued that the assured did not disclose this in the proposal, as he did not consider it a serious enough condition. Case 2 : A doctor applies for life insurance soon after a series of migrane attacks, which he does not disclose in the proposal papers. (A) The first is a case of innocent nondisclosure, while the second is one of deliberate disclosure. (B) The first is a case of innocent misrepresentation and the second one of deliberate misrepresentation. a) b) c) d) Both statements A & B are correct Statement A is correct Statement B is correct None of the above

343. What would be the paid-up value and surrender value as on 20.03.2006 for the data given below? Date of Commencement of Cover: 20.03.1995 Plan Term: Endowment 20 years with profit Sum Assured: Rs. 20,000 Mode: Half-yearly Bonus Accrued: Rs. 800 per thousand SA for 10 years duration SVF: 51.2% a) Rs. Rs. b) Rs. Rs. c) Rs. Rs. d) Rs. Rs. 27500 (Paid-up value) and 14080 (Surrender Value) 15,500 (Paid-up value) and 13,050 (Surrender Value) 25,000 (Paid-up value) and 12,800 (Surrender Value) 26,000 (Paid-up value) and 13,568 (Surrender Value)

341. Consider the following two cases: Case 1 : Famous paintings were destroyed in a fire accident in an art gallery. Case 2 : A cargo ship carrying 1000 ton grain was partially ruined by dampness while in transit. In both cases the items were insured as valued policies (1) The claims under both instances would be settled for the full value at risk.

Workbook

Risk Management and Insurance Planning

37

344. From the following data, calculate the amount of claim payable, if the insured dies on 27.10.2005. Date of Commencement: 13.09.1989 Sum Assured: Rs. 1,00,000 Plan Term: Money back (with profit) 25 years Last premium paid: 13.09.2004 Three installments of Rs. 20,000 each have been paid in 1994, 1999 and 2004 Assume that bonus declared is Rs.600 per thousand SA. Assume premium: Rs. 4,000 per year a) b) c) d) Rs. Rs. Rs. Rs. 1,00,000 64,000 1,24,000 1,56,000

Age 26: Rs. 52.00


a) b) c) d) Rs. 6,575 Rs. 7,775 Rs. 7,575 Rs. 6,775

347. Calculate surrender value: SA: Rs. 1,00,000 Date Of Commencement: 28.03.1985 Last Premium Paid: 28.03.1999 Term: 30 years Bonus: Rs. 80 per thousand SA Surrender Value Factor: 72% (for 14 years) 78% (for 15 years) 80% (for 16 years) Mode of Payment: Quarterly Date of Calculation: 28.06.1999 a) b) c) d) 83,916 82,620 82,600 114,800

345. Calculate the premium from the following data: Plan: Money Back 20 years Sum Assured: Rs. 1,00,000; rebate for large sum assured Rs. 2 per thousand Mode: Half-yearly; rebate - 1.5% Age: 36 years Proposal accepted at ordinary rates with double accident benefit Tabular premium: Rs. 69.25, Premium for double accident benefit is Re. 1 per thousand per annum a) b) c) d) Rs. 3,361 Rs. 3,461 Rs. 3,463 Rs. 3,261

348. From the following data, calculate the loan amount that can be given: Date of Commencement: 20.08.1990 Plan Term: Endowment with profit, 25 years Sum Assured: Rs. 1,50,000 Last premium paid 20.08.2003 Surrender Value Factor 55% Bonus Accrued Rs. 1,000/1,000 a) b) c) d) Rs. Rs. Rs. Rs. 1,16,000 1,19,900 1,20,500 1,61,700

346. Calculate half-yearly premium as on 15.10.2006 : SA: Rs. 3,00,000 Date Of Birth: 16.06.1982 Date of Maturity: 15.09.2036 Term: 25 years Double Accident Benefit: Re. 1 per thousand Mode Half-yearly Tabular premium Age 24: Rs. 49.50 Age 25: Rs. 50.60

349. From the following data, calculate the yearly installment: Sum Assured: Rs. 1,00,000 Plan Term: Endowment Plan 20 years Date of Maturity: 23.12.2022 Date of Birth: 28.06.1974 Tabular premium Age 27: Rs. 26.93 per thousand Age 28: Rs. 27.83 per thousand Age 29: Rs. 28.37 per thousand Premium Adjustment: 5% extra for monthly mode Re. 1.00 less for half-yearly mode Rs. 1.50 less for yearly mode

38

Risk Management and Insurance Planning

Workbook

Rebate of Rs. 1.50 for sum assured of Rs. 50,000 and above Assume premium will be paid annually a) b) c) d) Rs. 1,241 Rs. 1,242 Rs. 2,483 Rs. 2,843

354. Shah & Sons had a Fidelity Guarantee cover for its cashiers. The cashier committed a fraud in April 2002. The policy expired in the month of March 2003. Neither was it renewed. Fraud was detected in December 2003. Claim was lodged. The claim is ______. a) Payable b) Not Payable c) Payable partially 355. A Prospect and Planner agreed for an Endowment Assurance Policy for Rs. 4 Lakhs, limited payment for 10 years with the term of 20 years. If Reversionary bonus is taken as 7.5% p.a and terminal bonus as Rs. 150/-per 1000/-. The maturity value would be ______. a) b) c) d) Rs. 10,50,000 Rs. 10,60,000 Rs. 10,70,000 Rs. 10,80,000

350. Financial Quantifiable Loss is termed as _____________. a) b) c) d) Speculative Risk Financial Risk Personal Risk Monetary Risk

351. The direct advantage of insurance is a) b) c) d) Indemnification for unexpected losses Indemnification for expected profits Safety of funds Elimination of occurrence of insured risk

352. Mr. Shah took a Life Insurance Policy, which he continued for 3 years. He died within a night thereafter, suffering from cancer. Mrs. Shah lodged the claim as Nominee. The sum assured was Rs. 10 Lakhs. The insurer had indisputable evidence that Mr. Shah was hospitalized for 20 days earlier to his taking the proposal which was not disclosed in the proposal form. Though no fraudulent intention was established, the insurer repudiated the claim: _________________. a) Repudiation is in keeping with Law. b) Repudiation is not in keeping with the Law. 353. Malhotra Heart Centre has a great marketing centre and a full business. Dr. Kohli, the Chief Heart Surgeon is held in high esteem nationwide for his expertise and success rate in surgery. He has made an outstanding contribution for the last 8 years to enable market leadership for Malhotra Heart Centre. The most appropriate policy that the owners may take in the interests of business on the life of Dr. Kohli would be ________. a) b) c) d) Personal Accidental Insurance Term Insurance Key Man Insurance Mediclaim Insurance

356. Mr. Khanna will retire 6 months from now completing 65 years. His 2 daughters are residing abroad. He and his wife both have good health and live in a flat. Liabilities are insignificant and post retirement benefits substantial. The flat is very well decorated with valuable movable and immovable items. He purchased a car last year. He does not have any insurance cover except for the car, as of now. The most relevant cover for him would be __________. a) b) c) d) Endowment Assurance Money Back Household Insurance Term Insurance

357. The correct priority sequence in steps of review are_________. 1. Establish a frequency. 2. Establish change in clients circumstances. 3. Develop revised program. 4. Take approval of the client to revised program a) b) c) d) 1234 1342 1324 1423

Workbook

Risk Management and Insurance Planning

39

358. The advantage of insurance mechanism include ____________ a) b) c) d) Indemnification for unexpected losses Indemnification for expected profits Safety of funds Elimination of chance occurrence of insured risk

363. Who normally makes the offer in case of Insurance? a) b) c) d) Agent Insurer Prospect Anyone

364. The insurance contract promises to ______ a) Compensate in case of insured contingency b) Pay in case of any event c) Pay on demand d) Pay the premiums 365. The insurance contract is between ______ a) Insurance company and any person. b) Insurance company and its agent c) Insurance agent and the Insured client. d) Registered insurer and the insured (individual or firm) who is competent to contract. 366. What time does Insurable interest exist, in the case of property Insurance? a) b) c) d) At the time of entering contract At the time of loss occurring At any time Both at the time of entering into the contract and at the time of loss

359. Himanshu owns a flat worth Rs. 2,00,000 (market value). He has insured it only for Rs. 1,50,000. The flat is damaged by an earthquake, and loss is assessed at Rs. 1,00,000. Assuming the insurer applies the principle of average to the loss the insurer will get a claim payment of _________. a) b) c) d) Rs. 75,000 Rs. 70, 000 Rs. 37, 900 Rs. 1,00,000

360. The principles of Risk Management are applicable to ___________. a) b) c) d) Individuals Small Enterprises Huge Commercial enterprises All the above

361. The two elements involved in evaluation of risks includes probability ___________. a) of loss producing events occurring and potential losses b) of occurrence of events. c) of rising inflation on future income and cost of assets. d) of increasing cost of operation and employment 362. What is risk control? a) Anticipation, ascertaining and increasing the chances of occurrence of loss producing events b) Avoiding, eliminating and reducing the chances of occurrence of all events. c) Avoiding, eliminating and reducing the chances of occurrence of loss producing events or d) Anticipating, asserting and increasing the chances of occurrence of loss.

367. Insurable interest is not satisfied between whom under the Life Insurance contract? a) b) c) d) Husband-Wife Parent-Child Friends Employer and Employee

368. What does a Pure Risk involve? a) b) c) d) Loss or No Loss Loss and Gain Gain All the above

369. How much amount can the insurer recover under the existing subrogation rights? a) b) c) d) Equivalent to the claim. Equivalent to the sum insured Not more than 1 Lakhs 20 % of sum insured

40

Risk Management and Insurance Planning

Workbook

370. Material fact has been defined as the fact that would : a) Influence the mind of insured b) Influence the mind of underwriter in declining or accepting the risk c) Influence the mind of other policyholders d) Not affect the insurance contract 371. An example of material fact under Life insurance is __________ a) Occupation, health and habits of the person to be insured b) Particulars possessions of the person c) Particulars about the vehicle owned d) None of the above 372. Insurance : a) Protects the loss of an asset due to peril b) Compensates the loss caused by a peril c) Safeguards the loss of an asset due to peril d) None of the above 373. Violation of Utmost Good faith gives the aggrieved party: a) b) c) d) The right to avoid the contract The right to nondisclosure of breach The right to honour the contract None of the above

376. Insurance means : a) b) c) d) Sharing of risks Sharing of liability Sharing of responsibility None of the above

377. Life insurance is primarily a / an ________. a) b) c) d) investment tool financial security tool tax saving tool None of the above

378. _____________ are insurable. a) b) c) d) Speculative Risk Financial Risk Gambling Risk Pure Risk

379. The insurance mechanism is used to ____ a) b) c) d) control the risk transfer the risk retain the risk all of the above

380. Insurance covers : a) b) c) d) Dynamic Risk Speculative Risk Pure Risk Fundamental Risk

381. Peril refers to : a) b) c) d) Wear and tear of assets Natural calamities Accidental occurrences Either (b) or (c)

374. The term used to alter the insurance contract is _________. a) b) c) d) Deductibles Policies Endorsements All of the above.

382. The mechanism of insurance is ________ a) Reimbursement of losses by the Government b) Channeling of savings into investments c) Spreading losses by the community to minimize impact on a single member d) None of the above 383. Material facts should be disclosed during __________ a) b) c) d) Proposal stage Revival stage Alteration Any of the above

375. In co-insurance, the claims of the policy is shared by : a) All insurers in the same proportion as their share bears to the total value insured b) Insured and insurers c) Total value insured will be shared by main insurer d) Main insurer and the insured

Workbook

Risk Management and Insurance Planning

41

384. Uberrima fides means__________ a) b) c) d) Utmost Good Faith Ad idem Caveat Emptor Doctrine of warranty

392. The premium charged by the life insurer is calculated on the basis of __________ a) b) c) d) Mortality Rate Interest Expenses All of the above

385. Without insurable interest, an insurance contract would be ___________ a) b) c) d) Void-ab-initio Wagering contract Void contract Commercial contract

393. Which of the following are types of annuity? a) b) c) d) Immediate Deferred Flexible All of the above

386. Which of these is not an example of automatic insurable interest? a) b) c) d) Husband and wife Brother and sister Company and a key-executive Any of the above

394. What is Reinsurance? a) Distribution of insurance business made by the insured. b) An agreement concluded by one insurer with another. c) The other name given to coinsurance d) The action of insured in getting the same risk insured by another insurer. 395. Name the area(s) under which legal liability risks may arise. a) b) c) d) Under Statute Under Contract At Common Law All of the above.

387. Which material facts need not be disclosed? a) b) c) d) Facts of law Facts of common knowledge Facts known only to the proposer Both (a) and (b)

388. In order to be insurable, the risk should be: a) Capable of financial measurement b) Similar to a large number of homogenous risks c) Not against public policy d) All of the above 389. The components of Personal Risk that are faced by the individual is/are a) b) c) d) Death Disability Medical All the above

396. The damages that are awarded during personal injury claims include ________ a) Special damages and General damages b) Special damages only c) Damages to third parties only d) None of the above 397. What is the liability that arise due to failure of one of the parties not fulfilling the provisions of contract which it had entered into with another party? a) b) c) d) Vicarious Liability Special Damages Contractual Liability Strict Liability

390. A material fact is : a) A fact that would influence the judgment of a prudent insurer in fixing the premium b) A matter of common knowledge c) A fact that will reduce the premium d) A fact that will increase the premium 391. In case of whole life insurance, the sum assured is normally paid to the _______ a) b) c) d) Life assurer Nominee Legal heir Any of the above

398. Life Insurance is __________ a) b) c) d) Total loss cover Indemnity cover Substitute for loss of income All of the above

42

Risk Management and Insurance Planning

Workbook

399. Human Life value of an individual includes Future earnings of ____________ a) b) c) d) Self Self and dependents Family (Economic unit) Only dependents

Statement B: In a joint life policy, death claim is payable only when both the insured die. a) b) c) d) Only statement A Only statement B Both statements Neither of the statements

400. Insurable Risks are categorized into: a) Private and Official Risk b) Personal and Property and Liability Risks c) Official and Property Risks d) Private, Official and Property Risks 401. Human Life Value in general is _________ a) b) c) d) Total income of an individual Total wealth of an individual Earning capacity of an individual None of the above

406. A persons insurable interest in his own life is ___________ a) b) c) d) Unlimited 10 times his salary Equal to his assets Obligation for family

407. Which of the following statement(s) is/are True? Statement A: Sick people may be considered for grant of insurance on special terms. Statement B: Insurance cover begins as soon as the premium is received in the office along with the proposal. a) b) c) d) Only statement A Only statement B Both statements Neither of the statements

402. Human Life Value amounts to _________ a) Total savings b) Total investments c) Estimated future economic value of an individual d) Past income 403. Which of the following statements is/are True? Statement A: The fact of having been on a sick leave must be disclosed even if there were no sickness and the certificate of sickness was false. Statement B: Most plans of insurance are a combination of two basic plans. a) b) c) d) Only statement A Only statement B Both statements Neither of the statements

408. The behaviour of an agent who tells his client that the advice given by another agent is wrong is _______ a) b) c) d) Unprofessional Interfering Childish Dominating

409. What risk is covered by an annuity? a) b) c) d) Sickness Accident Living too long Death

404. Bringing a policy, which has lapsed, back to force is called___________ a) b) c) d) Recovery Renewal Revival Relapse

410. What measures are taken to reduce or eliminate the chances of occurrence of loss? a) b) c) d) Control Measures Cost Measures Administrative Measures All the above

405. Which of the following statement(s) is are True? Statement A: Life insurance policies are available through the Unit Trust of India also.

Workbook

Risk Management and Insurance Planning

43

411. The impact of the death of one sole breadwinner of the family over 40s family would be severe in the area of _______ a) b) c) d) Meeting Meeting Meeting Meeting outstanding debts basic needs medical needs educational needs

redemption and plans to sue Vikram for giving him misleading advice. Which of these methods can Vikram adopt as recourse? a) b) c) d) Asset Insurance Legal Liability Insurance Comprehensive Insurance Professional negligence cover

412. XYZ Ltd has a project that is insured for 10 crores. 60% of its risk is covered by Insurance Company A while 40% of the balance risk is covered by Insurance Company B. The insurance policy is issued by Insurance Company A. It has a collective insurance clause setting out the liability of both the insurance companies. This case is one of a) b) c) d) Co-insurance Reinsurance Both (a) and (b) None of the above

416. Victor is 24 years old and a young executive who has begun his career. He plans to start a family in 3-4 years. He travels extensively by road and out of the city. What kind of life insurance cover would you recommend for him? a) b) c) d) Endowment insurance policy Money back policy Unit linked policy Temporary insurance policy with convertibility and renewability option

413. Rahul and Priya (husband and wife) were born in 1950 and 1958 respectively. Rahuls life expectancy at birth is 75 years while Priyas life expectancy at birth is 72 years. Rahul is planning to buy an annuity that will be paid to him or his wife till any one of them is alive. For how long should he purchase this annuity if he is buying the same in the year 2008? a) b) c) d) 10 years 12 years 17 years 22 years

417. Mr. Sharma takes out a policy for his son Rahuls higher educational needs after Rahul reaches 18 years of age. Unfortunately, after paying a premium for 5 years, Mr. Sharma passes away. How can the policy be enforced without paying future premiums so that the intended benefits are preserved? a) b) c) d) Accident benefit premium Line on policy Premium waiver facility None of the above

414. Venkat proposes an insurance cover for his farmhouse which is occupied at intervals during the year. Which of these may be excluded from coverage by the insurance company? a) b) c) d) Fire Theft Third party liability Loss of rental income

418. Mr. Virendra Tripathi is an executive in an Indian company. He is in his early 30s and has a baby girl. He plans to sign up for a flat immediately. His spouse is not working. Mr. Tripathi has no life insurance yet. Which of these combinations would you recommend for his case? a) Limited payment endowment assurance, Joint life, Childrens deferred, Health b) Term insurance, Joint life, Childrens deferred, Mortgage redemption c) Whole life with term assurance for spouse, Educational annuity with premium waive d) Money back policy, Childrens deferred policy, Joint life policy, Mortgage redemption

415. Vikram, an insurance advisor advises his client, Mr. Singh about the purchase of insurance plans for tax exemptions which will earn Mr. Singh a 50% rebate. However, Mr. Singh was unable to obtain this tax

44

Risk Management and Insurance Planning

Workbook

419. The principle of insurance is based on ____________ a) b) c) d) Sharing of Resources Sharing of Losses Sharing of Capital Sharing of Investments

422. Read the following two statements: (I) Death of the breadwinner is a Financial Risk (II) It is a Pure Risk a) b) c) d) I & II are correct I & II are incorrect I is correct II is correct

420. Read the following data about life expectancy at birth of men and women during the last three decades in India-Census. Year 1971 1981 1991 2001 Male 46.40 54.10 58.10 62.30 Female 44.70 54.70 58.60 66.27

423. Ram lost his entire lifes savings when the owners of the private financial company, where he deposited it, went bankrupt due to questionable practices and defaulted on repayment. It is the case of loss under ___________. a) b) c) d) Investment Risk Speculative Risk Regulatory Risk None of the above

Certain conclusions are drawn below based on this study. Which of these conclusions cannot be deduced from this data? a) Today, female children enjoy longer lives than those born three decades ago b) Womens mortality has improved more than the mortality experience of men during the last c) The same trend as above is seen in the last two decades as well d) The mortality experience of men has improved in comparison to women in the last decade 421. Consider the following situations: 1. Mr. Sinha, aged 30, is a Major in the Indian Armed forces. His proposal and family history reveal that he suffered from typhoid at the age of 18 years. His father passed away a year ago at the age of 70 due to a heart attack. The underwriter will not consider any of these features as adverse. Mr. Bajaj, aged 35, is overweight by 10% and consumes one litre of beer every alternate day. His parents are alive. The underwriter is likely to ignore these features. Only Statement A is correct Only Statement B is correct Both statement A and B are correct Both statement A and B are incorrect

424. Rena had a choice of investing her retirement dues in post office savings or bank deposit or with private fund raiser. She barely manages to get her funds back when she opted for the third option. This is the case of ___________. a) b) c) d) Not understandings the risks Taking a gamble for high returns Succumbing to greed Exercising poor judgment

425. Mr. Jain has suffered several damages to his property (uninsured) because of fire. Thereby he suffered several financial losses. This happened because he did not cover _________. a) b) c) d) The Investment plan The Pure Risk The Speculative Risk None of the above

2.

a) b) c) d)

426. Tarun, a driver, caused injuries to a pedestrian by rash driving of car. The injured victim had to spend Rs. 1000 in treating his injuries. Taruns act has created liabilities under _________. a) b) c) d) Common Law Contract Statute and Common Law Statue only

Workbook

Risk Management and Insurance Planning

45

427. Mr. Roy took a business premises on lease with the provision that he himself had to pay the insurance premium for fire and other perils and not the owner of the premises. This would be an instance of __________. a) b) c) d) Risk Control Risk Transfer Risk Retention Risk Reduction

the space was rented out as a clothing store. The insurance company was not informed about the change of the tenant. The grounds on which the insurer refuses to entertain the claim, if the premises suffer a fire would be __________. a) b) c) d) Doctrine of utmost good faith Doctrine of adhesion Principle of duty of disclosure Principle of indemnity

428. Read the following : Case 1: An insurer receives an application for insurance which contains an incomplete or missing answer. The insurer does not contact the applicant for the missing information and the policy is issued, subsequently when claim arose under the policy, payment was denied on the basis of the earlier incomplete information. Case 2: In another case an insureds car insurance renewable premium was about to fall due. On enquiry with the company, they told him that there was 15 days grace period from that day during which time period he could pay. Subsequently, the car met with an accident next day, the company denied the claim on the ground that the premium was not paid on time. (A) The principle of estoppel and that of waiver in the first and second case respectively makes the denial of claim untenable. (B) In the former, the principle of waiver and in the latter, the principle of estoppel were violated, because of which the denial of claims were set aside by courts. a) b) c) d) A is correct B is correct A & B are cases of aleatory contact None of the above

431. Showroom of Rajesh & Sons, Drycleaners caught fire in an accident where many items of clothing of customers were destroyed. The contents were insured. The owners claim for damage of clothes etc. will be considered by the insurance company under _________. a) b) c) d) Property Liability Financial Liability Legal Liability None of the above

432. Which of the following contracts do not adhere to the application of law contract? a) b) c) d) Insurance Stock Market Property Deals None of the Above

433. Insurance contracts can be _____ contracts. a) b) c) d) Indemnity Negotiated None of the above Either a) and b)

434. Peter has insured his two-wheeler for a property risk. He wears a helmet while he drives but does not have an Accident Insurance. He has ______________. a) taken a grave risk by not taking accidental insurance. b) made a mistake, he should have taken Accidental Insurance instead of property risk insurance. c) transferred his property risk, controlled some of the risks and retained the rest. d) done a good thing by reducing his premium payout personal by not taking Accidental Insurance.

429. An insurable loss must not be ________ a) b) c) d) Catastrophic Fortuitous or accidental Definite None of the above

430. A fire insurance covered premises was housing a constructors office. In between, the constructor vacated the premises and

46

Risk Management and Insurance Planning

Workbook

435. Mark is an advocate having a roaring practice consisting of highly valued intricate cases of companies. He has 3 young juniors. Cash transactions do not take place in office. He maintains his office with modest furnishings. The most necessary cover for him is ______ a) b) c) d) Fidelity Guarantee Professional Indemnity Public liability Burglary

440. Sam got some injuries at the building site when the building contractors car ran over his toe. He was unable to attend work for 6 weeks. What are the options available to Sam? a) Sam and the contractor get the Accident Insurance. b) Sam gets Accident Insurance. The contractor gets third party Liability Insurance c) Sam gets Disability Insurance, if he has one. The Contractor gets a third party Liability Insurance d) Sam gets Disability Insurance, and the contractor gets Accident Insurance. 441. A Manager earning Rs. 40,50,000 p.a has the following details: Electronic items market value 3, Rs. 75,000 Acquisition value Rs. 37,500 Furniture replacement cost Rs. 21,00,000 House market value Rs. 97,50,000 Cost of building house Rs. 22,50,000 + cost of the land Daughters marriage Rs. 3,75,000 Childrens education Rs. 2,25,000 Retirement funding Rs. 6,75,000 Current professional Indemnity Rs. 2,25,000 Current investments 15,00,000 What should be his priority in regards of insurance to the following? 1. 2. 3. 4. a) b) c) d) PTD Life cover Property insurance Professional Indemnity 1, 2, 3, 4 2, 3, 1, 4 3, 1, 2, 4 4, 3, 1, 2

436. An insurance party reinsures the risk with a re-insurer, this happens in the case of _____ a) b) c) d) Risk Retention Risk Transfer Both a) and b) None of the above

437. Mr. Manish insures his home worth Rs. 50 Lakhs for Rs. 30 Lakhs. The house was destroyed in fire and he suffers loss worth Rs. 20 Lakhs. How much will he receive from the Insurance Company? a) b) c) d) 20 Lakhs 16 Lakhs 12 Lakhs 30 Lakhs

438. An insurable loss must be _________ a) b) c) d) Certain Fortuitous or accidental Indefinite Impossible to happen

439. Mr. Mohammed, 30 years and married, works for a firm which provides him with medical cover. He already has his own home and savings of Rs. 42 Lakhs which are well invested. In the next twenty years, he will be able to save enough to fund his retirement and his childrens education. Which of the following might be the most important insurance for him? a) b) c) d) Medical Cover Temporary Total Disablement Cover Property insurance Life Cover

Workbook

Risk Management and Insurance Planning

47

442. Mr. & Mrs. Sharma, aged 50 and 47, both have a life expectancy of 35 years. Calculate the insurance required based on need based and income replacement methods on Mr. Sharmas life. You have the following information: Current investments Rs. 25,00,000 Expenses Rs. 3,00,000 (including 1 Lakh of Mr. Sharmas personal expense) Mr. Sharmas income post tax Rs. 3.5 Lakhs Final costs Rs.1 Lakh Post tax, post inflation rate/discount factor is 3% a) b) c) d) Rs. 42 Lakhs, Rs. 117 Lakhs Rs. 20 Lakhs, Rs. 68 Lakhs Rs. 42 Lakhs, Rs. 83 Lakhs Rs. 20 Lakhs,Rs. 59 Lakhs

446. Mr. Dutta lives in Kerela, which is not earthquake prone. He decides to take out insurance on his property. As an insurance advisor, you would advise him that _______. a) He should not take out property insurance since he does not need it. b) He should not take the earthquake option, since it will be expensive c) He should not take out property insurance, since it will be expensive d) He can take out property insurance as the premium will depend on the risk and will therefore be cheaper 447. An insurance company makes a surplus, due to __________. 1. 2. 3. 4. 5. a) b) c) d) Mortality of policy Investment rate Expenses Lapse, surrenders Adverse loss ratio 1,2, 3 1,2,3,4 1,2,3,4,5 None of the above

443. Mr. & Mrs. Arora have Mr. Aroras father staying with them, who is entirely dependent on them. As an insurance advisor, you would estimate the life expectancy of _____. a) Mr. Arora because he manages the expenses. b) Mrs. Arora because if she is not there, who would look after the father. c) Mr. Aroras father because Mr. & Mrs. Arora have to support him for life. d) None of the above 444. Puneet, aged 50, works for a private firm. Most of his earnings is through commission. He does not have benefits like sick leave, LTA etc., he is single and already had TPD cover. His priority should be ____________. a) b) c) d) Life cover Health insurance Endowment policy Property insurance

448. Because of the application of the doctrine of adhesion, the courts construe the terms of insurance contract in favor of _______. a) b) c) d) The insured The insurer None of the above Depends on circumstances

449. Consider the following statements: (A) Payment of the first premium is legal consideration (B) Payment of the first years premium as also the subsequent two years premium constitute legal consideration. a) b) c) d) Only Statement A is correct Only Statement B is correct None of the above. Statements A and B are both incorrect

445. Insured Declared Value (IDV) does not apply to _________. a) b) c) d) Property damage in motor accident Curios, old jewellery Third party liability claim Property insurance

48

Risk Management and Insurance Planning

Workbook

450. Consider the following statements: (A) The proposer has to observe utmost good faith while proposing for insurance (B) Mere good faith is enough. a) Statements A & B both are correct b) Statement A & B both are incorrect c) Statement A is incorrect, while B is correct d) Statement A is correct & B is incorrect 451. Risks must be __________ before they can be _________ a) b) c) d) severe, assessed identified , measured measured, identified none of the above

455. Mr. Rajan has an outstanding debt of Rs. 2 Lakhs, medical expenses of Rs. 0.50 Lakhs per annum, education expenses of Rs. 2 Lakhs per annum and ot her self maintenance expenses of Rs.1 Lakh per annum plus he earns an annual income of Rs. 7 Lakhs, As a Financial Planner find out his Insurance Requirement as per Income Rule (Note :- A Five time income multiplier is the accepted industry norm). a) b) c) d) e) Rs. 7,50,000 Rs. 12,50,000 Rs. 10,00,000 Rs. 35,00,000 None of the above

456. Refer to the case above, as a Financial Planner find out his Insurance Requirement as per Income Plus Expense Approach a) b) c) d) e) Rs. 35,00,000 Rs. 39,50,000 Rs. 40,50,000 Rs. 41,00,000 None of the above

452. There is an Excess Clause of Rs. 5000 in a health insurance policy. So when a genuine claim is filed for Rs. 50000, In such case the insurance company will pay _________. a) b) c) d) e) Rs. 5000 Rs. 45000 Rs. 50000 Rs. 55000 None of the above

457. To whom the principles of risk management are applicable? a) b) c) d) Individuals Small enterprises Huge commercial enterprises All the above

453. A person has expenses relating to a loan of Rs. 2 Lakhs, medical expenses of Rs. 50000, education expenses of Rs. 2 Lakhs and other domestic expenses relating to Rs. 1 Lakh. With an annual income of Rs. 7 Lakhs and a 5 time income multiple the insurance need according to the income method is _____ a) b) c) d) Rs. Rs. Rs. Rs. 450000 500000 700000 3500000

458. Which are the two elements involved in evaluation of risks? a) Probabilities of loss producing events occurring and potential losses b) Probabilities of occurrence of events c) Probability of rising inflation on future income and cost of assets d) Probability of increasing cost of operation and employment 459. Which of the following is not an example of risk reduction when driving? a) b) c) d) Wearing a seat belt Driving within speed limits Driving defensively Not driving

454. In the above case the requirement as per the income plus expense method is ____ a) b) c) d) Rs. Rs. Rs. Rs. 3000000 3500000 4050000 4500000

Workbook

Risk Management and Insurance Planning

49

460. Which of the following is/are considered to be pure risk? a) b) c) d) Property risk Liability risk Risk from failure of others All the above

465. When evaluating a loss exposure, all of the following projections are made EXCEPT: a) Monetary value of a specific loss b) Total dollar loss of all losses during a particular time period. c) Frequency of losses during a particular time period. d) Location of the loss. 466. Which of the following statements concerning range is/are correct? I. Range equals the highest value minus the lowest value. II. Uses all values in the data set. a) b) c) d) I only II only Both I and II Neither I nor II

461. Rama owns a property worth Rs. 45 Lakh. She could only afford to pay a premium, which can insure her property to the extent of Rs. 35 Lakh. This is an example of _______________ a) b) c) d) Normal Insurance Over Insurance Double Insurance Under Insurance

462. The formula used to calculate the sum payable to the insured where a subject to average clause is mentioned in the contract is: __________________ a) Loss x (Sum Insured/ actual value) = sum payable b) Actual Value x (Sum Insured/ loss) = sum payable c) Sum payable/(loss x sum Insured) = Actual Value d) All the above 463. Ram got an Insurance from a company, which stood in his place in relation to a claim on a 3rd party. This example best relates to: a) b) c) d) utmost good faith un-insurable interest subrogation non-indemnity

467. Jack was very bored on a rainy day. He spent most of the afternoon randomly drawing cards to see how often an ace appeared. He drew a total of 1,050 cards. During that time, an ace appeared 84 times. What is the relative frequency of Jacks drawing an ace? a) b) c) d) 1.25% 8% 12.5% 15%

468. Which of the following is an advantage of pre-funding a loss? a) Money is free for other uses until needed for losses b) Costs can be spread over a period of time and limited to the exact amount required to pay for the losses c) Money in reserves cannot be used to support other activities d) Reserves can be built up over many budget periods 469. The human life value approach is based on _________ a) a persons eligibility for social security income b) the amount of life insurance he or she owns c) the standard of living he or she wishes to have d) the capitalized present value of potential lifetime earnings

464. Raj has insured his house with a well known insurer. He did not inform the insurance company that he has stored fire-works in his house. In this case he has failed to disclose a _________ a) b) c) d) relevant fact material fact known fact common fact

50

Risk Management and Insurance Planning

Workbook

470. Which of the following is a characteristic of an ordinary life policy? a) It is the most expensive form of cash value insurance. b) It matures at age 65. c) It has an increasing cash value and decreasing risk amount. d) Both the cash value and amount at risk increase annually. 471. Which of the following statements concerning yearly renewable term is correct? a) The insurance company may experience some adverse selection at renewal time. b) The premiums do not increase from year to year. c) Evidence of insurability must be furnished at the time of each renewal. d) The insured can renew the policy each year by completing a medical exam. 472. In case of insurance who should normally make the offer? a) b) c) d) agent insurer prospect anyone

476. A warranty in insurance contracts are: a) Guarantee b) Stipulations imposed by insurer because he wants to ensure that the risk remains same throughout the contract and does not increase c) Guarantees to increase the risk in case of default on part of insurer d) Guarantee to pay the loss to the insurer in case of the loss being higher than the sum insured. 477. What are used when the terms of insurance contracts are to be altered? a) b) c) d) deductibles endorsements policies all the above

478. __________ are the provisions under insurance contracts that stipulate deduction of a pre-specified amount from the claims payable under the policy a) b) c) d) endorsement rules deductibles none of the above

473. What is the binding force of any contract? a) b) c) d) offer acceptance parties consideration

479. ___________ is a compulsory deductible from each and every claim that the insurer pays to the insured a) b) c) d) premiums excess premiums and excess franchise

474. What is consideration that insurer makes under insurance contract? a) promise to compensate in case of insured contingency b) promise to pay in case of any event c) promise to pay on demand d) promise to pay the premiums 475. Which insurance contracts are not contracts of indemnity? a) b) c) d) Fire Marine Life Insurance Motor

480. A condition that increases the chance of loss is called a/ (n) a) b) c) d) Direct loss. Peril. Indirect (consequential) loss. Hazard.

Workbook

Risk Management and Insurance Planning

51

481. A publishing company solicits manuscripts for publication. The publishing company is concerned that an author might plagiarize material and that the person who was plagiarized might sue the publisher. To address this risk, the contract with the author includes a hold-harmless agreement. Through this agreement, the author, rather than the publisher, is held liable for plagiarism. In this situation, the publisher is using the hold-harmless agreement as what type of risk treatment measure? a) b) c) d) Risk selection Risk avoidance Risk transfer Risk retention

average loss is substituted for actual loss, is known as _____________ a) b) c) d) Fortuitous loss. Pooling of losses. Transfer of risk. Loss indemnification.

486. All of the following are ideal requirements that must be met for a risk to be privately insurable EXCEPT a) There must be a large number of similar exposure units. b) The chance of loss must be calculable. c) Losses should not be catastrophic. d) The loss should be within the insureds control. 487. Which of the following statements is (are) true with respect to insurance and gambling? I. Insurance is simply a wager that if a loss occurs, youll be paid for the loss. II. Both gambling and insurance are socially productive. a) b) c) d) Both I and II II only Neither I nor II I only

482. A risk that affects the entire economy, or a large number of persons or groups within the economy, is called a/ (n) a) b) c) d) Objective risk. Particular risk. Speculative risk. Fundamental risk.

483. All of the following are burdens of risk on society EXCEPT a) Risk forces individuals to practice loss control. b) Risk requires reserve funds to be setaside in case a loss occurs. c) Risk creates fear and worry. d) Risk deprives society of certain goods and services. 484. Deans Discount Store has been experiencing problems with shoplifting losses. Dean decided to install a camera monitoring system and to use magnetic price tags on products. If a tag is not demagnetized before the product bearing the tag leaves the store, an alarm bell sounds. These measures are examples of ___________ a) b) c) d) Risk transfer. Loss control. Risk avoidance. Risk retention.

488. Chaya does not own health insurance. For the past two weeks, she has been experiencing sharp abdominal pain. Given her condition, she would like to purchase health insurance. When higher-thanaverage risks (like Chaya) are insured at average premiums, losses are higher than anticipated. What is this problem called? a) b) c) d) Adverse selection Morale hazard Speculation Moral hazard

489. The first step in the risk management process is to _________________ a) Implement and administer the program. b) Select the appropriate techniques for handling losses. c) Identify potential losses. d) Evaluate potential losses.

485. The spreading of losses incurred by a few individuals over a larger group, so that

52

Risk Management and Insurance Planning

Workbook

490. All of the following risk treatment techniques are classified as risk control methods EXCEPT : a) b) c) d) Avoidance. Loss prevention. Loss indemnify. Loss reduction.

b) High insurance premiums and tight underwriting standards c) Low insurance premiums and tight underwriting standards d) High insurance premiums and loose underwriting standards 495. Jonathan believes there is a relationship between the number of miles driven by his companys delivery vehicles and the number of physical damage claims that will occur. Jonathan collected data on the number of claims and the number of miles driven for the past 15 years. Using a computer, Jonathan determined the coordinates of a line that best fit these data. Armed with this information, he can predict how many losses will occur next year. The type of analysis that Jonathan employed is called ____________ a) b) c) d) Regression analysis. Probability analysis. Cash flow analysis. Time value of money analysis.

491. Which of the following statements is (are) true with respect to the objectives of risk management? I. A pre-loss objective of risk management is the reduction of uncertainty. II. A post-loss objective of risk management is the stabilization of earnings. a) b) c) d) Neither I nor II S Both I and II I only II only

492. Bajaj Company manufactures electronic components. Managers of the company are considering several diversification options. One possibility is production of prescription drugs. When Bajaj Company managers learned of the potential legal liability that could result from the manufacture and sale of prescription drugs, the managers rejected the idea and decided to consider other diversification options. How did Bajaj Company choose to deal with the risk of legal liability arising from the manufacture and sale of prescription drugs? a) b) c) d) Loss control Risk transfer Risk retention Risk avoidance

496. In analyzing the probability that certain events will occur, a risk manager knows that certain events cannot occur together because the occurrence of the first event precludes the occurrence of the second event. Such events are called __________ a) b) c) d) Mutually exclusive events. Unconditional events. Dependent events. Independent events.

493. All of the following are methods used to pay retained losses EXCEPT : a) b) c) d) Borrowed funds. Insurance. Current net income. Funded reserve.

497. Olivia is Risk Manager of ABC Company. She is trying to determine if a loss control investment is justified. Olivia calculated the present value of the future cash flows she expects the project will generate. She summed the present value of the future cash flows and then subtracted the cost of the loss control equipment. The resulting value is called the projects _____________ a) b) c) d) Capital budget. Net present value. Internal rate of return. Loss distribution.

494. Which of the following is most likely to occur in a hard insurance market? a) Low insurance premiums and loose underwriting standards

Workbook

Risk Management and Insurance Planning

53

498. All of the following are benefits of insurance to society EXCEPT a) Provides a pool of investment funds. b) Enhances credit. c) Reduces insurance companys operating expenses. d) Indemnifies losses. 499. Kavita opened a boutique. After a fire damaged the boutique, Kavita was forced to close the business for four weeks while repairs were completed. The loss of profits that could have been earned if the business had remained open is best described as a/ an _________________ a) b) c) d) Hazard. Peril. Indirect (consequential) loss. Direct loss.

II. If the endorsement or rider conflicts with terms in the underlying contract, the endorsement or rider takes precedence unless it conflicts with the law. a) b) c) d) I only Both I and II Neither I nor II II only

504. Jitendra has an expected income stream over the next 15 years at Rs. 25 Lakhs. If the inflation adjusted discount rate comes to 3% then what is the human life value when there is a lump sum requirement of another 3 Lakhs to clear off his outstanding debt. (Assume the income stream per annum will remain at the same level for the next 15 years) a) b) c) d) Rs. Rs. Rs. Rs. 1304654 1604654 2984500 2204654

500. An insurance contract must be accepted in its entirety and any ambiguity in the contract is construed against the insurer. Because of these characteristics, we can describe insurance contracts as a) b) c) d) Aleatory contracts. Contracts of adhesion. Bilateral contracts. Valued contracts.

505. Which one of the following does not come under principal merit rating method? a) b) c) d) Schedule rating Experience rating Retrospective rating Judgement rating

501. In which part of an insurance contract would you find information about the property or activity to be insured? a) b) c) d) Conditions Insuring agreement Declarations Exclusions

506. As per the Income Tax Act, 1961 if Annual Premium paid is more than 20% of Sum Assured, then i. Such Premium is fully eligible for deduction u/s 80 C upto maximum of Rs. 1 Lakh and the Maturity Value, Surrender Value is fully exempt u/s 10(10D). ii. Such Premium is eligible for deduction u/s 80 C upto maximum of Rs. 1 Lakh but only to the extent of the first 20% of the Sum Assured. a) Both i & ii are false b) Only ii is False c) None of the above

502. Deductibles are used for all of the following reasons EXCEPT a) b) c) d) To eliminate small claims. To reduce premiums. To reduce loss control efforts. To reduce moral and morale hazard

503. Which of the following statements is (are) true with respect to endorsements and riders? I. Endorsements and riders are used to amend provisions of insurance contracts.

54

Risk Management and Insurance Planning

Workbook

507. M/s XYZ Ltd is entering the Capital Market to raise fresh funds(IPO) for its expansion, this is an example of ____________ from M/s XYZ Ltd point of view. a) b) c) d) e) f) Risk Retention Risk Reduction Risk Sharing Risk Transfer A combination of all above None of the above

512. Mr. A aged 22 years has just started working in a BPO earning Rs. 20,000 per month and has no dependants and is not financially responsible to his family. As a Financial Planner recommend the appropriate Insurance Plan. a) Term Insurance b) Term Insurance with Personal Accident and Disability Insurance c) Only Personal Accident and Disability Insurance d) No Insurance required, he should invest all his savings in Wealth Creation 513. Mr. Bhavesh aged 32 years is the sole breadwinner. He has a dependant spouse and a child named Asha who is 3 years old. As a Financial Planner recommend the appropriate Insurance Plan. (Note Asha will complete her MBA by her 25 age) a) Term Insurance with Critical Illness Rider, Disability Rider & Waiver of Premium Rider for the next 22 years b) Term Insurance with Critical Illness Rider, Disability Rider with Waiver of Premium for the next 25 years c) ULIP d) Money Back Policy for 25 years e) Endowment Policy for 22 years f) No Insurance required, he should invest all his savings in Wealth Creation for Asha MBA Education g) None of the above 514. Mr. Chitale, a practicing Chartered Accountant aged 43 years provides Advisory Services relating to Mergers & Acquisitions, Corporate Debt Restructuring, Capital Financing. As a Financial Planner recommend the Priority Insurance Plan. (Note: Mr Chitale will retire at 60 age) a) Term Insurance with Critical Illness Rider, Disability Rider & Waiver of Premium Rider for the next 17 years b) ULIP c) Money Back Policy for 17 years d) Endowment Policy for 17 years e) Professional Indemnity Insurance f) None of the above

508. Which of the following is the third step in Risk Management Strategy a) Identification of the Risks b) Develop alternative strategy for handling risks c) Choose and implement an appropriate Strategy. d) Analyze and evaluate risks (Risk Measurements) e) None of the these 509. As per Sec 10(10D) of the Income Tax Act, any sum received from Insurance Company is exempt fully for Income Tax. a) True b) False c) True only when Annual Premium paid is less than or equal to 20% of Sum Assured d) True for Death Benefit without any reference to Annual Premium condition e) Both c) & d) are true 510. Premium paid to Life Insurance for Basic Sum Assured (Death Benefit) is eligible for deduction under which section of the Income Tax Act, 1961. a) Sec 80 C upto a Maximum of Rs. 1 Lakh b) Sec 80 C without any Maximum limit c) Sec 80 D d) None of the above 511. Premium paid to Life Insurance for Value Additions such as Critical Illness Rider is eligible for deduction under which section of the Income Tax Act, 1961. a) b) c) d) e) Sec 80 C upto a Maximum of Rs. 1 Lakh Sec 80 C without any Maximum limit Sec 80 D upto a Maximum of Rs.15000 Sec 80 D without any Maximum limit None of the above

Workbook

Risk Management and Insurance Planning

55

515. As per the Insurance Act, 1938 an Insurance Company has the right to question the claim on the basis of concealment of facts, misstatements and other information in the proposal form, such right is covered under which clause? a) Incontestability Clause u/s 45 covering only the first two years form the policy commencement date b) Incontestability Clause u/s 45 covering only the first three years form the policy commencement date c) Incontestability Clause u/s 45 covering only the first five years form the policy commencement date d) Refusal of Claim clause u/s 45 covering only the first two years form the policy commencement date e) None of the above 516. Mr. Rajesh is worried about his childs (Sunita aged 5 years) education expenses and he does not want to stop her education due to lack of funds. As a Financial Planner recommend the Insurance Plan which will meets the need of security for childrens education. a) Term assurance plan covering period till the child completes her Professional Education b) Child Education Plan with waiver of premium order c) Deferred annuity plan d) Health insurance e) ULIP f) None of the above 517. Industrial Plant Insurance Policy has a Franchise Clause of Rs. 50,000, the claim filed is Rs. 70,000. What would be the claim amount payable by the insurance company under the said policy? a) b) c) d) e) Rs. 50,000 Rs. 70,000 Rs. 20,000 Rs. 1,20,000 None of the above

amount payable by the insurance company under the said policy? a) b) c) d) e) Rs. 30,000 Rs. 29,000 Rs. 59,000 Rs. Nil None of the above

519. In Mediclaim, what is the maximum amount of premium which is deductible under the Income Tax Act, 1961? a) Rs. 10,000, only if paid by a cheque Sec 80 DD b) Rs. 15,000, only if paid by cheque Sec 80 D c) Rs. 15,000, only if paid by a cheque Sec 80 DD d) Rs. 10,000, only if paid by a cheque Sec 80 D e) None of the above 520. The immediate cash needs of an individual will come to Rs. 30000 for the funeral (final cost) plus Rs. 3 Lakhs as outstanding debt. The Net Income needs at a present value are Rs. 4 Lakhs. With an existing asset base of Rs. 2 Lakhs, compute the insurance required under Needs Approach. a) b) c) d) e) Rs. 330000 Rs. 730000 Rs. 530000 Rs. 930000 None of the above

521. An Insurance planner and prospect agreed for an endowment policy for Rs. 8 Lakhs limited premium payment for 10 years with a term of 20 years. If revisionary bonus is taken as 7.5 % per annum and terminal bonus as Rs. 150 per Rs. 1000 sum assured, what will be the Maturity value? a) b) c) d) e) Rs. 15,20,000 Rs. 21,20,000 Rs. 13,20,000 Rs. 20,00,000 None of the above

518. Boiler Plant Insurance Policy has a Franchise Clause of Rs. 30,000, the claim filed is Rs. 29,000. What would be the claim

56

Risk Management and Insurance Planning

Workbook

522. The following are statements made concerning contracts of insurance. Identify the statement/s that is/are correct. (i) For life insurance contracts, misstatement of the insureds age constitutes a voidable misrepresentation. (ii) An innocent misrepresentation by an applicant for insurance constitutes fraud. a) b) c) d) only (i) is correct only (ii) is correct Both (i) & (ii) are correct Neither (i) or (ii) are correct

526. Mr. Ankush is in very good health without any background of existing diseases, he needs a temporary Assurance Cover. As a Financial Planner recommend an appropriate insurer. a) Insurer with a stringent underwriting policy b) Insurer with a lax underwriting policy c) Insurer with very few causes of death excluded d) Insurer with extensive causes of death excluded e) None of the above 527. Identify the class of assurance for which an insurer, in a costing exercise, must provide for future anticipated improvements in mortality. a) investment-linked endowment assurance b) conventional endowment assurance c) group life assurance d) life annuities e) none of the above 528. Mr. Suresh has come with the following risk matrix, which he is exposed to. As a Financial Planner advise him an appropriate Risk Management Strategy.

523. The maxim buy term and invest the balance may not be a feasible proposition for many prospects for the following reasons EXCEPT: a) it may be difficult to achieve sufficient diversification in the invested assets b) it may be difficult to achieve a suitable investment portfolio with the desired risk reward relationship c) it may be possible to consistently outperform the investment returns earned by an established life office d) that the insurer offers capital guarantees on cash values e) None of the above 524. An individual may prefer investment-linked assurance to conventional assurance policies for the following reasons EXCEPT: a) he has some direction over the investment of his premiums b) he is informed of the expenses charged for the services provided c) he is attracted by the guaranteed surrender values offered under these contracts d) he prefers the switching facilities available under such contracts e) None of the above 525. Which of the following does not constitute a valid charge on the premiums paid for an investment-linked life assurance policy? a) b) c) d) e) bid-offer spread front-end charges recurrent fund related charges surrender penalty None of the above

a) Risk Transfer, Risk Avoidance, Risk Retention & Risk Reduction b) Risk Avoidance, Risk Control, Risk Retention & Risk Reduction c) Risk Transfer, Risk Retention, Risk Avoidance & Risk Reduction d) Risk Transfer, Risk Reduction, Risk Avoidance & Risk Retention e) None of the above

Workbook

Risk Management and Insurance Planning

57

529. Professional Indemnity Insurance policy protects a financial planner who has been negligent in giving investment advice _____ a) Only if the financial planner is liable under statute b) Only if a contract exists between the financial planner and the investor c) Only if the investor relies on the advice d) Only if the financial planner has not included a disclaimer clause in the contract with the investor e) Only if the financial planner is a Qualified Certified Financial Planner f) None of the above 530. Consideration under the law is a written promise to ________ (i) Do certain things (ii) Abstain from doing certain things (iii) Forbear some acts (iv) Accept an offer a) b) c) d) e) (i), (ii), and (iii) only (i), (ii), and (iv) only (i), (iii), and (iv only (ii), (iii), and (iv) only None of the above

532. Mrs. Parab explains that she needs a life insurance policy that will cover her family against financial risk over the next five years, while she still has dependent children and a large home mortgage. It is unlikely her income will increase over the covered tenure. As a Financial Planner advise her about her Insurance Policy. a) Children Benefit Policy for 5 years b) Term insurance with a level premium for 5 years c) Single premium Money Back Policy maturing after 5 years d) Mortgage Redemption Insurance for 5 years e) Whole of life assurance to be made paid up after five years f) ULIP to be redeemed after 5 years g) None of the above 533. Mr. Sanjay aged 59, presently in his retirement phase, has maintained a high level of life insurance, the reason may be due to ________ (i) Pay estate duty (ii) Provide succession or dependent income (iii) Repay outstanding debt (iv) Build cash value a) b) c) d) e) Only (i) and (ii) Only (iii) Only (i), (ii) and (iii) All of the above None of the above

531. Mr. Frank has an Indemnity Policy in respect of his owned house. The house was valued when he first took out the policy three years ago at Rs. 80,000 and insured for that amount. The policy has been renewed each year since, without any alteration or modifications. In the current year, the said house was completely destroyed by fire and the cost of rebuilding it was Rs. 110,000 (Market Value) plus he paid Rs. 35,000 to his neighbour whose house was accidentally burnt to some extent due to the fire. As a Financial Planner compute his claim amount which will be paid by the Insurance Company. a) b) c) d) e) Rs. 58,182 Rs. 80,000 Rs. 93,182 Rs. 1,45,000 None of the above

534. Terminal Bonus is to be paid _________ a) compulsorily by the Insurance Company as per the Insurance Act, 1938. b) as a percentage of sum assured every year. c) by the Insurance Company voluntarily for being disciplined in regular premium payment d) None of the above

58

Risk Management and Insurance Planning

Workbook

535. Shelar & Suvarna are married couple who are 44 & 40 years respectively. They want a regular series of payments in their Sunset Years covering both of their lives. Shelars wish is that Suvarna should get the regular payment even if he becomes a departed soul, the same is the wish of Suvarna, that Shelar should get the regular payment even if she becomes a departed soul. As a Financial Planner address their concerns by giving them an appropriate advise, i.e. which annuity the couple should select? a) b) c) d) e) f) g) Life time Annuity Immediate Annuity Deferred Annuity Annuity Certain Annuity Certain with Life Time Annuity Joint-and-Survivor Annuity None of the above

form if he wishes to make a Single payment (Ignore impact of Taxes) a) Rs. 1488.07 b) Rs. 1458.25 c) Rs. 1460.25 d) Rs. 1898.75 e) None of the above 538. Mr. Ramesh aged 20 , wants a regular series of payments of Rs. 1,200 per annum in accrued form from his age 55 without any interruption where he want its certain for the first 10 years and thereafter if he survives he wants the same till he becomes a departed soul. To achieve the aforesaid retirement goal he is ready to invest in a single payment a maximum of Rs. 2500 now in accrued form, Mr. Ramesh left for heavenly abode on his 75th birthday. If the annual rate of earnings is taken as 6% per annum compounded annually throughout the working life & sunset years. As a Financial Planner find out the IRR(Ignore impact of Taxes) a) b) c) d) e) 4.22 % 2.22 % 3.22 % 2.82 % None of the above

536. Rupali purchased a Health Insurance. The policy has a calendar-year deductible of Rs. 500 and 80:20 co-insurance. Rupali was hospitalized with a covered illness on January 23rd. This hospitalization was her first claim under the said policy for the calendar year. Her covered medical expenses were Rs. 20,500. How much of this amount will the insurer pay, and how much will Rupali be required to pay to the Hospital? a) The insurer will pay Rs.1 6,500 Rupali will pay Rs. 3,500 b) The insurer will pay Rs. 20,500 Rupali will pay Rs. Nil c) The insurer will pay Rs. 15,500 Rupali will pay Rs. 4,500 d) The insurer will pay Rs. 20,000 Rupali will pay Rs. 500 and and and and

539. Mr. Shreekant aged 30 years took a Pure Risk Cover (Term Insurance) bearing a sum assured of Rs. 10 Lakhs for a term of 25 years costing an annual premium of Rs. 3,500 per annum. After paying the said premium for a period of 20 years well on time, he wants to surrender the policy. (Note:- Its the Insurance Company official standard to pay Surrender Value @ 75% of Accumulated Premium if the policy has been serviced for 19 years and more) As a Financial Planner find out the Surrender Value of the said policy if any? a) b) c) d) e) Rs. Nil Rs. 52500 Rs. 49,875 Rs. 70,000 None of the above

537. Mr. Praful aged 20 , wants a regular series of payments of Rs. 1,200 per annum in accrued form with an escalation of 2% per annum from his age 55 to next 20 years without any interruption. If the annual rate of earnings is taken as 8% per annum compounded annually throughout the working life & retirement life and an Inflation Rate of 5% per annum during his golden years. As a Financial Planner find what contribution he is required to make right now in accrued

Workbook

Risk Management and Insurance Planning

59

540. A person requires Rs. 50000 per annum for the 15 years after retirement, which is still 10 years away. With a discount rate of 5% the need for the insurance based on this parameter is __________ a) b) c) d) Rs. Rs. Rs. Rs. 245675 334541 456376 567645

543. The proceeds of Rs. 150,000 from a life insurance policy is to be disbursed under a set tlement option using an annuity accrued with term certain of 15 years. Assuming that a 6% per annum interest compounded annually is appropriate, as a Financial Planner find out the level annual payments arising under this arrangement a) b) c) d) e) Rs. 13,491 Rs. 12,972 Rs. 15,444 Rs. 14,570 None of the above

541. Maggi & Savio are co-applicants of a mortgaged house. They are on the verge of a divorce, however they are skeptical about the action that may be initiated by the Housing Finance Company (HFC) once they are divorced. As a Financial Planner address their concerns by giving them an appropriate advise about the action of the Home Loan Lender. a) HFC will not interfere as long as the EMIs are being paid on time b) HFC will repossess the house after divorce c) HFC will insist on the house being transferred to only one of them d) HFC will appoint a Counselor for reconciliation between the couple. e) HFC will increase the interest rate in order to compensate for the increased risk f) HFC will apply the Forfeiture Clause immediately on divorce g) None of the above 542. Edward a Roman Catholic got married on 25th December 2005 in the Marriage Court under Special Marriages Act. On 28 th December 2005 does his wife Anita a Hindu have an insurable interest in the life of Edward ? (Note :- Anita after Marriage wont get converted to Christianity) a) No, Insurable Interest is acquired only after a year of marriage b) Yes, Anita has an automatic insurable interest in the life of Edward soon on marriage and the likewise for Edward c) No, only Edward has an insurable interest on the life of the Anita and not the other way round. d) No, Insurable Interest is only among the same caste e) None of the above

544. The proceeds of Rs. 250,000 from a life insurance policy is to be disbursed under a set tlement option using an annuity accrued with term certain of 15 years. Assuming that a 8% per annum interest compounded monthly is appropriate, as a Financial Planner find out the level annual payments arising under this arrangement a) b) c) d) e) Rs. 2373 Rs. 2389 Rs. 27043 Rs. 29207 None of the above

545. Rupesh aged 33 and Sunita aged 29 (Married Couple) approach you a Financial Planner with the following data:Funds for investment are limited & needs are in ample as follows:1) To start an investment plan for funding the Education of their child Akshay, aged 3years. 2) To set up a Testamentary Trust for their child. 3) To set up a contingency fund amounting to 3 months of living expenses . 4) To start saving for retirement. 5) To purchase life and health insurance policies. As a Financial Planner address their concerns by giving them an appropriate advise by sequencing there needs in the order of priority. a) b) c) d) e) 3,5,1,4,2 5,1,2,4,3 5,3,1,4,2 3,5,2,1,4 None of the above

60

Risk Management and Insurance Planning

Workbook

546. Mr. Andrew recently purchased a 2 BHK Flat worth Rs. 30 Lakhs and wants to insure it, As a Financial advise him that the flat can be insured by giving the following reason ______ a) Insurable interest has been created Common Law b) Insurable interest has been created Contract c) Insurable interest has been created Statute d) Insurable interest has been created Ownership e) None of the Above f) All of the Above by by by by

Directions for questions 550 to 552. Refer to the following data. Mr. Praful has given his data as follows: Current Age 35 years. Will retire at 55 Retirement Fund : Required 2/3 of his last drawn Salary per annum in due form Life expectancy : 20 years after retirement His current annual salary is Rs. 60,000 and this escalates by 6% per annum throughout his working career, His Employer gives the annual escalations on his birthdays. Assume the rate of earning of 8% per annum compounded annually throughout the overall period.

547. In the event of a claim, an Insurance contract governed by Indemnification clause pays the insured the claim amount equal to ______ a) the sum insured b) the extent of loss admitted subject to a maximum equal to the sum insured c) the extent of loss admitted without any limits d) a previously agreed amount e) Negotiable amount between the Insured & Insurer f) None of the above 548. Mr. Sunil has an Endowment Life Insurance Policy where he has duly paid his premiums for the past 15 years out of a term of 20 years, he is in urgent needs of funds only for a short period in order to marry off his son Paras. As a Financial Planner advise him the maximum loan he can avail on the said policy? a) b) c) d) e) f) 85% of Surrender Value 90% of Paid up Value 90% of Surrender Value 90% of all premiums paid 75% of all premiums paid None of the above

550. Calculate his last drawn annual salary on the eve of retirement. a) b) c) d) e) Rs. 128,400 Rs. 181,536 Rs. 192,428 Rs. 132,000 None of the above

551. Calculate the present value of the retirement income as on the current age of 35 yrs. a) b) c) d) e) Rs. 2,25,800 Rs. 2,54,933 Rs. 2,75,328 Rs. 3,05,780 None of the above

552. Calculate the level annual amount that Praful must deposit during his working life, assuming that the first is due now, to enjoy the stated retirement income during his sunset years. a) b) c) d) e) Rs .28,042 Rs. 25,965 Rs. 24,042 Rs. 28,890 None of the above

549. The characteristics of Insurable Risks are: 1. 2. 3. 4. a) b) c) d) e) Large Number of Homogenous Units Definite and Measurable Fortuitous or accidental Not Catastrophic 1&2 2, 3 & 4 1,2 & 3 1,2,3 & 4 None of the above

Workbook

Risk Management and Insurance Planning

61

553. Participating ordinary life policies with a sum assured of Rs. 10,000 are issued by two Life Insurance Companies, LIC and I-Pru, to Suresh aged 35 with the following cost data:

555. LIC Premium Table depicts the following data for a certain policy in which your client Mr. Sanjay is interested Tabular premium: Rs. 33.10 per Rs. 1000 sum assured. Rs. 2 less for yearly mode. Re. 1 less for half yearly mode. Rs. 3 less for sum assured of Rs. 1, 00,000 and above. Double accident is allowed up to a maximum of Rs. 10 Lakhs sum assured on payment of Re. 1 per 1000 sum assured.

Based on the above information and assuming a 6% per annum interest rate, the annual surrender cost index for each Rs. 1,000 sum assured at the end of 20 years ___________ a) b) c) d) For LIC is greater than I-Pru For LIC is equal to I-Pru For I-Pru is greater than LIC Is not determinable, since data is insufficient e) None of the above 554. Ms. Kavita has an Endowment Life Insurance Policy where she has duly paid the premiums, she is in urgent needs of funds which she doesnt want to repay back in order to marry off her only daughter Sanjivani. As a Financial Planner advise her the maximum amount she can avail on the said policy on its due surrender. a) b) c) d) e) Date of commencement 5th October, 1990 Date of last premium 5th April, 2000 Date of birth 1st May, 1965 Mode of premium payment - Half Yearly Premium Rs. 20 per annum for Rs. 1000 sum assured Plan Endowment with profit Term 25 years Sum assured Rs. 1, 20,000 Bonus accrued from 31st March 1991 to 31st March 2000 is Rs. 70 per Rs. 1,000 sum assured Accidental benefit Rs.2 per annum for Rs. 1000 sum assured S.V. factor 75% of Accumulated Value Rs. 24000 Rs. 81,810 Rs. 80,810 Rs. 80,100 None of the above

As a Financial Planner calculate the yearly premium for Rs. 15 Lakhs sum assured with occupation extra of Rs. 4 per thousand sums assured a) b) c) d) e) Rs. 49,150 Rs. 48,150 Rs. 48,650 Rs. 52,350 None of the above

556. Mr. Jagadish has given his personal details as follows: Current Age 30 yrs , Plans to Retire at age 65. Job Profile : Senior Manager in Telco Ltd with Annual Salary of Rs. 10,00,000 Annual Cash Outflow a) b) c) d) e) Professional tax of Rs. 5,000 Income tax Rs. 1,95,000 Reasonable self-maintenance expenditure Rs. 1,00,000 p.a. Life insurance premium for self Insurance Rs. 20,000 having sum assured Rs. 20 Lakhs Life insurance premium for Sulekha (wife) Rs. 13000 having sum assured Rs. 5 Lakhs Life insurance premium for Aditya (son) Rs.7000 having sum assured Rs.2 Lakhs Assume:- Rat e of interest for capitalization of future income is at 10%. As a Financial Planner Calculate the Insurance Group using HLV Method Rs. 46 Lakhs Rs. 68 Lakhs Rs. 48 Lakhs Rs.6 6 Lakhs None of the above

62

Risk Management and Insurance Planning

Workbook

557. Mr. Shelar invests Rs. 5000 in a Bank Deposit today @ 8% p.a compounded monthly. He hopes that this investment will enable him to fund his college education (estimat ed t o cost Rs. 9000) which commences after 4years. As a Financial Planner compute the end value of this investment after four years? (Ignore the impact of Inflation & the rise in Education cost) a) b) c) d) e) Rs. 6802 Rs. 6870 Rs. 6878 Rs. 6925 None of the above

As a Financial Planner find out the Insurance Gap of Mr. Noronha as per Needs Approach & Income Replacement Approach (Ignore Impact of Inflation) a) b) c) d) e) Rs. 69.34 Lakhs, Rs. 83.33 Lakhs Rs. 49.34 Lakhs, Rs. 68 Lakhs Rs. 50.42 Lakhs, Rs. 65.33 Lakhs Rs. 70.34 Lakhs, Rs. 83.33 Lakhs. None of the above

560. Raman needs a life insurance policy for Rs. 5 Lakhs that will require a premium commitment of Rs. 23000 each year for the next 15 years. With an earning potential of 5% the adjusted interest earning potential comes to ________ a) b) c) d) Rs. Rs. Rs. Rs. 396406 521122 596406 696406

558. Kavita invests Rs. 5000 per year at the beginning of each year for 5 years @ 5% p.a. in a bank deposit. She then withdraws the accumulated sum over a period of 3 equal annual installments at the end of the said each period. As a Financial Planner compute the value of the deposit at the end of 5 years and the quantum of withdrawal each year thereafter? a) b) c) d) e) Rs. 27628, Rs. 10145 Rs. 29010, Rs. 10145 Rs. 29568, Rs. 11054 Rs. 29010, Rs. 10653 None of the above

559. Mr. and Mrs. Noronha, aged 40 and 36 years, both have a life expectancy of another 40 years. Their Data Sheet depicts the following information: Mr. Noronha is the sole breadwinner of the family and the family has no children Current investments has a Market Value of Rs. 20,00,000 Annual Expenses Rs.4 Lakhs (including Rs. 1 Lakh of Mr. Noronha personal Exps) Mr. Noronha income post tax Rs. 3.5 Lakhs. Final costs Rs. 1 Lakhs (Funeral Exps of Mr. Noronha). Post tax, post inflation rate/discount factor is 3 % per annum

Workbook

Risk Management and Insurance Planning

63

Answers
Question 1 2 3 4 5 6 7 8 9 10 11 12 13 14 15 16 17 18 19 20 21 22 23 24 25 26 27 28 29 30 31 32 Answer c c b b a d d a c c c d c b a b b c a a a c c b a b a d c d a b Question 33 34 35 36 37 38 39 40 41 42 43 44 45 46 47 48 49 50 51 52 53 54 55 56 57 58 59 60 61 62 63 64 Answer c d d b c c a d d b c c d a a a c d c b c b a b c b c b b c a a Question 65 66 67 68 69 70 71 72 73 74 75 76 77 78 79 80 81 82 83 84 85 86 87 88 89 90 91 92 93 94 95 96 Answer a c a c d d d a d c d c b a d a b c d a b c d a a c c a b c d b

64

Risk Management and Insurance Planning

Workbook

Answers
Question 97 98 99 100 101 102 103 104 105 106 107 108 109 110 111 112 113 114 115 116 117 118 119 120 121 122 123 124 125 126 127 128 Answer a b b c a d c d b a b a c d b c b c b c d c d b c b b a a c a c Question 129 130 131 132 133 134 135 136 137 138 139 140 141 142 143 144 145 146 147 148 149 150 151 152 153 154 155 156 157 158 159 160 Answer d d d a b c c c b c a c c b a b c d b d a d b c a a c c b c b c Question 161 162 163 164 165 166 167 168 169 170 171 172 173 174 175 176 177 178 179 180 181 182 183 184 185 186 187 188 189 190 191 192 Answer c c b c d a c b b a b d d a c d b b d c a a b c d c a b b b c b

Workbook

Risk Management and Insurance Planning

65

Answers
Question 193 194 195 196 197 198 199 200 201 202 203 204 205 206 207 208 209 210 211 212 213 214 215 216 217 218 219 220 221 222 223 224 Answer c d c d d a d a c d d a b b a b c a b c a c c b c c c b a a b a Question 225 226 227 228 229 230 231 232 233 234 235 236 237 238 239 240 241 242 243 244 245 246 247 248 249 250 251 252 253 254 255 256 Answer a a c a b b d b b a d c a c d d a a a a d c b a d c a a c d d d Question 257 258 259 260 261 262 263 264 265 266 267 268 269 270 271 272 273 274 275 276 277 278 279 280 281 282 283 284 285 286 287 288 Answer d d d a a d c d a a d d b d a a c a c a c a a b c d c b b c d a

66

Risk Management and Insurance Planning

Workbook

Answers
Question 289 290 291 292 293 294 295 296 297 298 299 300 301 302 303 304 305 306 307 308 309 310 311 312 313 314 315 316 317 318 319 320 Answer d d d d a b c c b d b c b a d c a a b a c d b d d a c d d b d d Question 321 322 323 324 325 326 327 328 329 330 331 332 333 334 335 336 337 338 339 340 341 342 343 344 345 346 347 348 349 350 351 352 Answer d b b b c d c b a d a d c a d a c a a c b a a c a c d d c b a b Question 353 354 355 356 357 358 359 360 361 362 363 364 365 366 367 368 369 370 371 372 373 374 375 376 377 378 379 380 381 382 383 384 Answer c b b c a a a d a c c a d d c a a b a b a c a a b d b c d c d a

Workbook

Risk Management and Insurance Planning

67

Answers
Question 385 386 387 388 389 390 391 392 393 394 395 396 397 398 399 400 401 402 403 404 405 406 407 408 409 410 411 412 413 414 416 416 Answer b b d d d a b d d b d a c c c b c c a c d a a a c a b a d b d d Question 417 418 419 420 421 422 423 424 425 426 427 428 429 430 431 432 433 434 435 436 437 438 439 440 441 442 443 444 445 446 447 448 Answer c c b d b a a a b c b b a c c d d c b b c b d b b d a b c d b a Question 449 450 451 452 453 454 455 456 457 458 459 460 461 462 463 464 465 466 467 468 469 470 471 472 473 474 475 476 477 478 479 480 Answer a d b b d c d c d a d d d a c b d a b b d c b c d a c b b c b d

68

Risk Management and Insurance Planning

Workbook

Answers
Question 481 482 483 484 485 486 487 488 489 490 491 492 493 494 495 496 497 498 499 500 501 502 503 504 505 506 507 508 509 510 511 512 Answer c d a b b d c a c c b d b b a a b c c b c c b c d b c b e a c c Question 513 514 515 516 517 518 519 520 521 522 523 524 525 526 527 528 529 530 531 532 533 534 535 536 537 538 539 540 541 542 543 544 Answer a e a b b d b b b d c c d a d a c a a b b c f d a b a b a b c b Question 545 546 547 548 549 550 551 552 553 554 555 556 557 558 559 560 Answer a d b c d b c b c e b a c d c b

Workbook

Risk Management and Insurance Planning

69

Explanatory Answers
1. A composite agent is one who is authorized to sell both Life & General insurance products. Ans, [c]. In property insurance one needs to have insurable interest both at the time of contract and los s. Exception is Marine insurance werein one needs to have insurable interest only at the time of Loss. Further in Life insurance one needs to have insurable interest only at the time of contract. Ans, [c]. Ans, [b]. Ans, [b]. Ans, [a]. Ans, [d]. Ans, [d]. Tort is civil wrong doing. Ans, [a]. Ans, [c]. Ans, [c]. Ans. [c]. Ans. [d]. Ans. [c]. Ans. [b]. Ans. [a]. Ans. [b]. In case of strict liability certain defenses are allowed but not so in Absolute liability. Ans. [b]. Ans. [c]. Professional Indemnity requirement as per IRDA norms: 24. It is loss of checked in baggage which is covered. Ans. [b]. Ans. [a]. Ans. [b]. Ans. [a]. Ans. [d]. Ans. [c]. Ans. [d]. Ans. [a]. Ans. [b]. Ans. [c]. Ans. [d]. Ans. [d]. Ans. [b]. Ans. [c]. Ans. [c]. Ans. [a]. Ans. [d]. Ans. [d]. Ans. [b]. Ans. [c]. Ans. [c]. Jan Arogya policy is a Medicalim / health insurance policy by United India Insurance Company. Ans. [d]. Intentional Tort could be Libel - Publication of false statement to damage others reputation. Slander - Damaging reputation verbally Assault - Unlawfully touching another person. Ans. [a]. Bhavishya Arogya is a life term policy where medical benefits are made available after retirement of the insured- by New India Assurance. Ans. [a]. Ans. [a]. Ans. [c]. Ans. [d]. Ans. [c].

25. 26. 27. 28. 29. 30. 31. 32. 33. 34. 35. 36. 37. 38. 39. 40. 41. 42. 43. 44. 45.

2.

3. 4. 5. 6. 7. 8. 9. 10. 11. 12. 13. 14. 15. 16. 17.

18. 19.

46.

47.

Ans. [a]. 20. 21. 22. 23. Ans. [a]. Ans. [a]. Ans. [c]. Ans. [c].

48. 49. 50. 51.

70

Risk Management and Insurance Planning

Workbook

52.

In property insurance one needs to have insurable interest both at the time of contract and loss. Exception is Marine insurance, where one needs to have insurable interest only at the time of Loss. Further in Life insurance one need to have insurable interest only at the time of contract. Ans. [b]. Ans. [c]. All are water related perils. Ans. [b]. Ans. [a]. Partic ular ris k effec ts an individual where as Fundamental risk effects whole economy. Ans. [b]. Ans. [c]. Ans. [b]. Ans. [c]. Ans. [b]. Ans. [b]. Ans. [c]. Ans. [a]. Ans. [a]. Ans. [a]. Ans. [c]. Ans. [a]. Ans. [c]. Ans. [d]. Ans. [d]. Ans. [d]. Ans. [a].

80.

Policy Cost (per thousand)

[PM + CSVP] [1 + i] [CSV + D] [DB CSV ] 0.001

53. 54.

PM = Premium-yearly; CSV= Cash surrender value at the end of policy year; CSVP = cash surrender value at the end of previous year; i = interest or rate of return; D = Dividend or bonus; DB = Death benefit; policy cost conversion given=0.001. Hence answer is 19.70. Ans. [a]. 81. Policy Cost (per thousand)

55. 56.

[PM + CSVP] [1 + i] [CSV + D] [DB CSV ] 0.001

57. 58. 59. 60. 61. 62. 63. 64. 65. 66. 67. 68. 69. 70. 71. 72. 73.

PM = Premium-yearly; CSV = Cash surrender value at the end of policy year; CSVP = cash surrender value at the end of previous year; i = interest or rate of return; D = Dividend or bonus; DB = Death benefit; policy cost conversion given=0.001. Hence answer is 44.66. Ans. [b]. 82. Policy Cost (per thousand)

[PM + CSVP] [1 + i] [CSV + D] [DB CSV ] 0.001

PM = Premium-yearly; CSV = Cash surrender value at the end of policy year; CSVP = cash surrender value at the end of previous year; i = interest or rate of return; D = Dividend or bonus; DB = Death benefit; policy cost conversion given=0.001. Hence answer is 22.62. Ans. [c]. 83. Policy Cost (per thousand)

[PM + CSVP] [1 + i] [CSV + D] [DB CSV ] 0.001

30 20 = The answer is 12 Lakhs. 50 Ans. [d].


Ans. [c]. Ans. [d]. Ans. [c]. Ans. [b]. Fundamental risk affects whole ec onomy and particular risk affects individual. Ans. [a]. Ans. [d]. 84.

74. 75. 76. 77. 78.

PM = Premium-yearly; CSV = Cash surrender value at the end of policy year; CSVP = cash surrender value at the end of previous year; i = interest or rate of return; D = Dividend or bonus; DB = Death benefit; policy cost conversion given=0.001. Hence answer is 12.29. Ans. [d]. Policy Cost (per thousand)

[PM + CSVP] [1 + i] [CSV + D] [DB CSV] 0.002

79.

PM= Premium-yearly; CSV = Cash surrender value at the end of policy year; CSVP = cash surrender value at the end of previous year; i = interest or rate of return; D = Dividend or bonus; DB = Death benefit; policy cost conversion given=0.002. Hence answer is 9.85. Ans. [a].

Workbook

Risk Management and Insurance Planning

71

85.

Policy Cost (per thousand)

92.

[PM + CSVP] [1 + i] [CSV + D] [DB CSV] 0.002

Apply the same concept as shown in Question 88. The answer is 1500. Ans. [a]. Apply the same concept as shown in Question 88. The answer is 1250. Ans. [b]. Apply the same concept as shown in Question 88. The answer is 1400. Ans. [c]. Apply the same concept as shown in Question 88. The answer is 2750. Ans. [d]. Salary = (55000 12) (3000 + 132000 + 45000 + 18000) = 462000/- PMT Working Life left = 60 30 = 30 years = NPER Rate of Capitalization = 8% pa = Rate HLV = 52 Lakhs (approx) Net recommended amount = 52 Lakhs 12 Lakhs = Rs. 40 Lakhs (approx) Ans. [b].

93.

PM = Premium-yearly; CSV = Cash surrender value at the end of policy year; CSVP = cash surrender value at the end of previous year; i = interest or rate of return; D = Dividend or bonus; DB = Death benefit; policy cost conversion given=0.002. Hence answer is 22.33. Ans. [b]. 86. Policy Cost (per thousand)

94.

95.

[PM + CSVP] [1 + i] [CSV + D] [DB CSV] 0.002

96.

M = Premium-yearly; CSV = Cash surrender value at the end of policy year; CSVP = cash surrender value at the end of previous year; i = interest or rate of return; D = Dividend or bonus; DB = Death benefit; policy cost conversion given=0.002. Hence answer is 11.31. Ans. [c]. 87. Policy Cost (per thousand)

[PM + CSVP] [1 + i] [CSV + D] [DB CSV] 0.002

97.

Rs. 15 Lakhs (5000 + 410000 + 55000 + 100000) = 930000 Rate is 5% Nper is 25 HLV = PV = 1.31 crore Hence the answer is Rs.2.26 crore. Ans. [a].

PM = Premium-yearly; CSV = Cash surrender value at the end of policy year; CSVP = cash surrender value at the end of previous year; i = interest or rate of return; D = Dividend or bonus; DB = Death benefit; policy cost conversion given=0.002. Hence answer is 6.14. Ans. [d]. 88. 30000 = 0.006 Mortality = 5000000 No. of people who would die = 50000 0.006 = 300 Amount required to pay them = 300 200000 = 6 crores 60000000 Premium = = 1200/50000 Hence the answer is 1200. Ans. [a]. 89. Apply the same concept as shown in Question 88. The answer is 625. Ans. [a]. Apply the same concept as shown in Question 88. The answer is 1750. Ans. [c]. Apply the same concept as shown in Question 88. The answer is 2000. Ans. [c].

98.

Revisionary Bonus = 30,000 20 = 6.00 Lakhs Terminal Bonus SA Maturity value 0.60 Lakhs 4.00 Lakhs 10.60 Lakhs

Hence the answer is Rs.10,60,000 Ans. [b]. 99. Ans. [b].

100. Ans. [c]. 101. Ans. [a]. 102. Ans. [d]. 103. Ans. [c]. 104. Ans. [d]. 105. Ans. [b]. 106. Indemnity is to bring back the insured back to the position he/she was prior to the loss. There should be no gain. Ans. [a].

90.

91.

72

Risk Management and Insurance Planning

Workbook

107. Ans. [b]. 108. Insurance companies can provide long term funds for the country which can be channeled in to long term infrastructure projects. Ans. [a]. 109. Material fact is any fact which would influence the insurer in accepting or declining a risk or in fixing the premium or terms and conditions of the contract. Here the colour of car is non material to the risk being insured. Ans. [c]. 110. Ans. [d]. 111. Ans. [b]. 112. Ans. [c]. 113. Ans. [b]. 114. Ans. [c]. 115. Ans. [b]. 116. Ans. [c]. 117. Ans. [d]. 118. Ans. [c]. 119. Ans. [d]. 120. Ans. [b]. 121. Ans. [c]. 122. In insurance contract , the insured makes offer and it is the company which accepts/denies. Further, if insurance company changes any terms it makes an counter offer to the insured and then it is up to insured to accept or deny. Ans. [b]. 123. Ans. [b]. 124. Ans. [a]. 125. Ans. [a]. 126. Rating could be 1. Individual or judgmental 2. Class or manual 3. Merit rating Schedule, experienc e and Retrospective. Ans. [c]. 127. Ans. [a]. 128. Ans. [c]. 129. Ans. [d]. 130. Landless Agricultural Labourers Group Insurance (LALGI) scheme is a social security group scheme where in head of the family between 18 to 60 years is eligible if has no land. On his death a sum of Rs.2000/ is paid to the family. Ans. [d].

131. The large number of units have to be homogenous. Ans. [d]. 132. Ans. [a]. 133. Ans. [b]. 134. Ans. [c]. 135. The property is under insured by 25 % and hence the claim would be of 75 % of Rs. 15 Lakhs = Rs. 1125000 Ans. [c]. 136. Insurance is for indemnification purpose and there cannot be any profit from it. Ans. [c]. 137. Ans. [b]. 138. Excess is a compulsory deductible from claims. Here 10 % and hence 2000 would be deducted. Ans. [c]. 139. The purpose of excess is to keep away the small claims and reduce the expenses of servicing claims. So, nothing would be paid here as excess is 5000 and los s is 4500 which is below exc ess limit. Ans. [a]. 140. Ans. [c]. 141. The claim cannot be more than the limit fixed in the policy. Ans. [c]. 142. Both the companies would contribute to the loss. Ans. [b]. 143. In insurance there is no scope for profit or gain, only indemnification. Ans. [a]. 144. Both the companies would contribute to the loss. Ans. [b]. 145. Ans. [c]. 146. Rate is the price per unit of coverage i e 1000 in insurance while premium is the total cost or price paid for the coverage. Ans. [d]. 147. Ans. [b]. 148. Ans. [d]. 149. Premium allocation charges are generally higher in the 1st year in ULIP plans. Ans. [a]. 150. Nominee of an insured commiting suicide within 1 year of taking policy is not eligible for the benefits. Ans. [d]. 151. Ans. [b]. 152. Ans. [c].

Workbook

Risk Management and Insurance Planning

73

153. Ans. [a]. 154. Ans. [a]. 155. Ans. [c]. 156. Ans. [c]. 157. Ans. [b]. 158. Ans. [c]. 159. Ans. [b]. 160. Ans. [c]. 161. Ans. [c]. 162. Ans. [c]. 163. Ans. [b]. 164. Ans. [c]. 165. Ans. [d]. 166. Ans. [a]. 167. Ans. [c]. 168. Ans. [b]. 169. Ans. [b]. 170. Ans. [a]. 171. Ans. [b]. 172. Ans. [d]. 173. Ans. [d]. 174. Ans. [a]. 175. Ans. [c]. 176. Ans. [d]. 177. Ans. [b]. 178. Ans. [b]. 179. Ans. [d]. 180. Ans. [c]. 181. The underwriter tries to manage the risk of adverse selection. C is incorrect, because the rate is usually set after establishing underwriting standards. D is incorrect, because the purpose is not to avoid exposure but to manage risks. B is a responsibility of underwriters but is not their main responsibility. Ans. [a]. 182. Ans. [a]. 183. Ans. [b]. 184. Ans. [c].

185. Use capital liquidation approach (i) Need Based: PMT : (5 1) = 2 Lakhs NPER : 40 Rate : 3% Type : 1 Pv = 47.61 20 = 27.61 Lakh Income approach PMT : (3.5 1) = 2.5 NPER : 40 Type : 1 Rate : 3% Pv : 59.52. Ans. [c]. 186. PMT = 45000 ; NPER = 20 years ; Rate = 6 %; Type - 1; PV= 5,47,115. Ans. [c]. 187. FV= 120000 ; NPER = 15; Rate = 6 %; PV= 50071. Ans. [a]. 188. 5,47,115 + 50,071 + 15,000 4,70,000 = 142186 = 143000. Ans. [b]. 189. Tabular premium payment mode discount large SA discount +occupation + DAB 53.40 0.80 = 52.60 2.00 = 50 .60 + 3 + 1 = 54.60 54.60 150 = Ans. [b]. 8190 = 4095. 2

190. Paid-up value No. of premiums paid sum assured / No. of premium payable + bonus if any. Surrender value paid up value S.V factor/100 Paid-up value = 9 1, 50,000/100 = Rs.13, 500 + 22, 500 = 36, 000 (As last premium paid was 13.02.92, the interim bonus on 31.03.92 is not taken for calculation loan amount) Surrender value = 36,000 52.3/100 = Rs.18, 828 Loan amount: 85% of surrender value, as it is paidup condition 85 = Rs.16, 003.8 or 16, 004 18, 828 100 150000 Paid up value = 9 + 22500 = 36000 100 52.3 SV = 36000 = 18,828 100 85 Loan 18828 = 16004. 100 85% because policy is though paid up but lapsed. Ans. [b].

74

Risk Management and Insurance Planning

Workbook

191. No. of premium paid sum assured/number of premium payable 21 Rs.1, 00,000 = 26, 250 80 Paid-up value Rs.26, 250. Ans. [c]. 192. Maturity Amount is 50000 2.5 = 1,25,000/Maturity Benefit = Basic SA + Reversionary Bonus + terminal bonus] 125000 = 50,000 + 35 50 15 + terminal bonus or terminal bonus = 48750/-. Ans. [b]. 193. Find rate given NPER = 15; PMT = 5066; FV = 125000; Rate = 6.76 %. Ans. [c]. 194. The question asks what is needed to calculate the value of a life. The human life value is determined by finding the present value of the future cash flows (the clients annual salary). (i), (ii), (iii) and (iv) are all needed in this calculation. Ans. [d]. 195. Ans. [c]. 196. Life insurance policies can be issued to anyone with an insurable interest. The policy may be assigned to anyone and anyone may be named as beneficiary. The insured need not be insurable at the time of assignment. Ans. [d]. 197. Ans. [d]. 198. Ans. [a]. 199. Ans. [d]. 200. Ans. [a]. 201. Ans. [c]. 202. Ans. [d]. 203. Ans. [d]. 204. Ans. [a]. 205. Ans. [b]. 206. Ans. [b]. 207. Ans. [a]. 208. Ans. [b]. 209. Ans. [c]. 210. Ans. [a]. 211. Ans. [b]. 212. Ans. [c]. 213. Ans. [a]. 214. Ans. [c].

215. Ans. [c]. 216. Ans. [b]. 217. Ans. [c]. 218. Ans. [c]. 219. Ans. [c]. 220. Excess is a compulsory deductible and hence 45000 would be paid. Ans. [b]. 221. Ans. [a]. 222. Ans. [a]. 223. Ans. [b]. 224. Ans. [a]. 225. Ans. [a]. 226. Ans. [a]. 227. Ans. [c]. 228. Ans. [a]. 229. 2,00,000 75% + 1,00,000 = 22.42,000 (approx). 7% Ans. [b]. 230. 30750 2500 = 28,250. Ans. [b]. 231. Rating could be I. Individual or judgmental II. Class or manual III. Merit rating Sc hedule, Ex perience & Retrospective Ans. [d]. 232. Bima kavach Yojna is a premium back term plan by Birla Sunlife specially for rural masses. Ans. [b]. 233. Jeevan Anand is combination of whole life and endowment, W hile all other 3 are money back policies. All these policies are from LIC. Ans. [b]. 234. Ans. [a]. 235. Rajarajeshwari Mahila Kalyan is a Disablity policy by United India Insurance Company. Ans. [d]. 236. Ans. [c]. 237. 150000 + 20000 = 170000.Because 2 Lakhs is income needed in retirement and not an immediate need. Ans. [a]. 238. 3 Lakhs + 12 Lakhs + 5 Lakhs = 20 Lakhs. Ans. [c].

Workbook

Risk Management and Insurance Planning

75

239. 20 Lakhs + 1.7 Lakhs 2.70 Lakhs 8 Lakhs = 11 Lakhs. Ans. [d]. 240. Only premium payable can be worked from the information and not the cover. Ans. [d]. 8 241. 5 + 1 + 2 = 8 % of income. 500000 = 40000. 100 Ans. [a]. 242. Ans. [a]. 4 , so for 25,000 one can buy for 6250000. 1000 Ans. [a]. 244. Concept of Annuity. The answer is option [a]. Ans. [a]. 245. Ans. [d]. 10% 246. Find PV given rate ; NPER = 12 5; 12 PMT = 1200; PV = 56478. Ans. [c]. 7 247. 50000 = 17500. 20 Ans. [b]. 248. Ans. [a]. 249. Ans. [d]. 250. Total loss = 2 20 Lakhs + 30 2 Lakhs = 100 Lakhs They are underinsured by 50 % so 2500 = 1000 Rate = 2.50. Ans. [c]. 251. 20000 10000 = 10000 1000 = 9000. Ans. [a]. 252. Refer to chapter 11 of the Risk Management and Insurance Planning concept book. Ans. [a]. 253. Surrender cost method = 79159 Ans. [c]. (79159 24400) 254. Net payment method = 34.719 1577 .28 = 100 Ans. [d]. [ 24400 + 35260] 561 .6325 = 34.719 100

255. Refer to chapter 11 of the Risk Management and Insurance Planning concept book. Ans. [d]. 256. Refer to chapter 11 of the Risk Management and Insurance Planning concept book. Ans. [d]. 257. Refer to chapter 11 of the Risk Management and Insurance Planning concept book. Ans. [d]. 258. Ans. [d].

243.

259. Ans. [d]. 260. Ans. [a]. 261. Ans. [a]. 262. Ans. [d]. 263. Ans. [c]. 264. Ans. [d]. 265. Ans. [a]. 266. Ans. [a]. 267. Ans. [d]. 268. Ans. [d]. 269. Ans. [b]. 270. Ans. [d]. 271. Ans. [a]. 272. Ans. [a]. 273. Ans. [c]. 274. Ans. [a]. 275. Ans. [c]. 276. Ans. [a]. 277. Ans. [c]. 278. Ans. [a]. 279. Ans. [a]. 280. Ans. [b]. 281. Ans. [c]. 282. Ans. [d]. 283. Ans. [c]. 284. Ans. [b]. 285. Ans. [b]. 286. Ans. [c]. 287. Ans. [d]. 288. Ans. [a]. 289. Ans. [d].

5000000 2000

76

Risk Management and Insurance Planning

Workbook

290. Ans. [d]. 291. Ans. [d]. 292. Ans. [d]. 293. Ans. [a]. 294. Ans. [b]. 295. Ans. [c]. 296. Ans. [c]. 297. Ans. [b]. 298. Ans. [d]. 299. Ans. [b]. 300. Ans. [c]. 301. Ans. [b]. 302. Ans. [a]. 303. Ans. [d]. 304. Ans. [c]. 305. Ans. [a]. 306. Ans. [a]. 307. Ans. [b]. 308. Ans. [a]. 309. Ans. [c]. 310. Ans. [d]. 311. Ans. [b]. 312. Ans. [d]. 313. Ans. [d]. 314. Ans. [a]. 315. Ans. [c]. 316. Ans. [d]. 317. Ans. [d]. 318. Ans. [b]. 319. Ans. [d]. 320. Ans. [d]. 321. Ans. [d]. 322. Ans. [b]. 323. Ans. [b]. 324. Ans. [b]. 325. Ans. [c]. 326. Ans. [d]. 327. Ans. [c]. 328. Ans. [b].

329. Ans. [a]. 330. Ans. [d]. 331. Ans. [a]. 332. Ans. [d]. 333. 75000 100 = 1071429 + 128572 = 1200001, 7

which is 12 times salary. Ans. [c]. 334. Because there is a 15 days cooling period where in one can ask for refund of premium by canceling the policy. Ans. [a]. 335. Ans. [d]. 336. Ans. [a]. 337. Ans. [c]. 338. Ans. [a]. 339. Ans. [a]. 340. Ans. [c]. 341. Ans. [b]. 342. 4820, 51.75 1.5525 = 50.1975 - 2 = 48.1975 per thousand for 1 Lakh = 48.20 x 100 = 4820 Ans. [a]. 23 = 11500 + 800 20 [bonus] = 27500 40

343. 20000

assuming hes paid March 06 premium unless stated otherwise. SV = 27500 Ans. [a]. 344. Premium payable from 13/9/89 to 13/9/2005 17 4000 = 68000 Bonus declared Less : Premium unpaid for 13.09.2005 600 100 = 60000 4000 124000 hes paid march 06 premium unless stated otherwise. Ans. [c]. 345. 69.25 1.5% 69.25 2.00 + 1 = 67.21 33.61. Ans. [a]. 346. 49.50 + 1 = Ans. [c]. 100 = 2 51 .2 = 14080. 100

50 .5 300 = 7575. 2

Workbook

Risk Management and Insurance Planning

77

347. (i)

57 1,00,000 = 47500 120 800 10 14 159 .50

371. Ans. [a]. 372. Ans. [b]. 373. Ans. [a]. 374. Ans. [c]. 375. Ans. [a]. 376. Ans. [a]. 377. Ans. [b]. 378. Ans. [d]. 379. Ans. [b]. 380. Ans. [c]. 381. Ans. [d]. 382. Ans. [c]. 383. Ans. [d]. 384. Ans. [a]. 385. Wagering is betting/gambling. Ans. [b]. 386. Ans. [b]. 387. Ans. [d]. 388. Ans. [d]. 389. Ans. [d]. 390. Ans. [a]. 391. Ans. [b]. 392. Ans. [d]. 393. Ans. [d]. 394. Ans. [b]. 395. Ans. [d]. 396. Ans. [a]. 397. Ans. [c]. 398. Ans. [c]. 399. Human Life Value (HLV) which computes value of an individual is a present value of own future earnings. Ans. [c]. 400. Ans. [b]. 401. Ans. [c]. 402. Ans. [c]. 403. Ans. [a]. 404. Ans. [c]. 405. Ans. [d]. 406. Ans. [a]. 407. Ans. [a].

(ii) Bonus

= 114840 = 72% 159.500. Ans. [d]. 57 348. (i) 1,00,000 = 47500 120 800 10 14 (ii) Bonus 159 .50 = 114840 = 72% 159.500. Ans. [d]. 349. 27.83 3 [1.5 discount for yearly mode + 1.5 SA discount]= 24.83 Ans. [c]. 350. Ans. [b]. 351. Ans. [a]. 352. Ans. [b]. 353. Ans. [c]. 354. Ans. [b]. 355. Revisionary Bonus = 30,000 * 20 = 6.00 Lakhs Terminal Bonus SA Maturity value Ans. [b]. 356. Ans. [c]. 357. Ans. [a]. 358. Ans. [a]. 359. Because under insured by 25%. Ans. [a]. 360. Ans. [d]. 361. Ans. [a]. 362. Ans. [c]. 363. Ans. [c]. 364. Ans. [a]. 365. Ans. [d]. 366. Ans. [d]. 367. Ans. [c]. 368. Ans. [a]. 369. Ans. [a]. 370. Ans. [b]. 0.60 Lakhs 4.00 Lakhs 10.60 Lakhs

78

Risk Management and Insurance Planning

Workbook

408. Ans. [a]. 409. Ans. [c]. 410. Ans. [a]. 411. Ans. [b]. 412. Ans. [a]. 413. Ans. [d]. 414. Ans. [b]. 415. Ans. [d]. 416. Ans. [d]. 417. Ans. [c]. 418. Ans. [c]. 419. Ans. [b]. 420. Ans. [d]. 421. Ans. [b]. 422. Ans. [a]. 423. Ans. [a]. 424. Ans. [a]. 425. Ans. [b]. 426. Ans. [c]. 427. Ans. [b]. 428. Ans. [b]. 429. Ans. [a]. 430. Ans. [c]. 431. Ans. [c]. 432. Ans. [d]. 433. Ans. [d]. 434. Ans. [c]. 435. Ans. [b]. 436. Ans. [b]. 437. Ans. [c]. 438. Ans. [b]. 439. Ans. [d]. 440. Ans. [b]. 441. Ans. [b]. 442. Need based : PMT = 200000; NPER = 35; rate = 3 % Type = 1; PV= 44.26 Lakhs. add 1 Lakh final cost and deduct 25 Lakhs investments 44.26 + 1 25 = 20.26 Lakhs

income replacement [retention] = 2,50,000 = 83.333 Lakhs + 1 25 = 59 Ans. [d]. 443. Ans. [a]. 444. Ans. [b]. 445. Ans. [c]. 446. Ans. [d]. 447. Ans. [b]. 448. Ans. [a]. 449. Ans. [a]. 450. Ans. [d]. 451. Ans. [b]. 452. Ans. [b].

100 3

453. 5 7 Lakhs = 35 Lakhs, which is Income multiple method. Ans. [d]. 454. Ans. [c]. 455. Ans. [d]. 456. Ans. [c]. 457. Ans. [d]. 458. Ans. [a]. 459. Ans. [d]. 460. Ans. [d]. 461. Ans. [d]. 462. Ans. [a]. 463. Ans. [c]. 464. Ans. [b]. 465. Ans. [d]. 466. Ans. [a]. 467. Ans. [b]. 468. Ans. [b]. 469. Ans. [d]. 470. Ans. [c]. 471. Ans. [b]. 472. Ans. [c]. 473. Ans. [d]. 474. Ans. [a]. 475. Ans. [c]. 476. Ans. [b]. 477. Ans. [b].

Workbook

Risk Management and Insurance Planning

79

478. Ans. [c]. 479. Ans. [b]. 480. Ans. [d]. 481. Ans. [c]. 482. Ans. [d]. 483. Ans. [a]. 484. Ans. [b]. 485. Ans. [b]. 486. Ans. [d]. 487. Ans. [c]. 488. Ans. [a]. 489. Ans. [c]. 490. Ans. [c]. 491. Ans. [b]. 492. Ans. [d]. 493. Ans. [b]. 494. Ans. [b]. 495. Ans. [a]. 496. Ans. [a]. 497. Ans. [b]. 498. Ans. [c]. 499. Ans. [c]. 500. Ans. [b]. 501. Ans. [c]. 502. Ans. [c]. 503. Ans. [b]. 504. PMT = 25 Lakhs; NPER = 15; Rate = 3%; Loan is immaterial. Ans. [c]. 505. Rating could be 1. Individual or judgmental 2. Class or manual 3. Merit rating Sc hedule, Ex perience & Retrospective Ans. [d]. 506. Ans. [b]. 507. Ans. [c]. 508. Ans. [b]. 509. Ans. [e]. 510. Ans. [a]. 511. Ans. [c].

512. Ans. [c]. 513. Ans. [a]. 514. Ans. [e]. 515. Ans. [a]. 516. Ans. [b]. 517. Ans. [b]. 518. Franchise is a type of deductible where in claim above franchise limit is fully paid and below which nothing is paid. Ans. [d]. 519. Franchise is a type of deductible where in claim above franchise limit is fully paid and below which nothing is paid. Ans. [b]. 520. 4 + 3 + 0.30 2 = 7.30 Lakhs Ans. [b]. 521. Revisionary bonus = 800000 = 60000 20 100 = 1200000 Terminal bonus = 150 800 = 120000 Maturity value = 8 + 12 + 1.2 = 21.2 Lakhs Ans. [b]. 522. Ans. [d]. 523. Ans. [c]. 524. Ans. [c]. 525. Ans. [d]. 526. Ans. [a]. 527. Ans. [d]. 528. Ans. [a]. 529. Ans. [c]. 530. Ans. [a]. 80000 = 58182. 531. 80000 110000 Ans. [a]. 532. Ans. [b]. 533. Ans. [b]. 534. Ans. [c]. 535. Ans. [f]. 536. Ans. [d]. 537. Inflation adjusted return = 1.08 % 1.07 = 93457. PV = 22001.69 (assumed money at begun) FV = 22001; Rate = 8%; NPER = 35; PV = 1488.07 Ans. [a].

7 .5

80

Risk Management and Insurance Planning

Workbook

538. Find FV given Rate = 6%; NPER = 35; PV = 2500; FV = 19215 Ans. [b]. 539. Ans. [a]. 540. Find PV given PMT = 50000; Rate = 5%; NPER = 15; now FV = 544932; Rate = 5%; NPER= 10; PV = 334541 Ans. [b]. 541. Ans. [a]. 542. Ans. [b]. 543. Find PMT given PV = 150000; Rate 6 %; NPER = 15 Ans. [c]. 8 544. Find PMT given PV = 250000; Rate = and 12 NPER 15 12 Ans. [b]. 545. Ans. [a]. 546. Ans. [d]. 547. Ans. [b]. 548. Ans. [c]. 549. Ans. [d]. 550. Find FV given PV = 60000; Rate = 6%; NPER = 19; FV = 181536 Ans. [b]. 551. 181536

557. Find FV given PV = 5000; Rate 8% /12 and NPER 48. Ans. [c]. 558. Find FV given Rate= 5 %; NPER = 5; PMT= 5000; Type =1; FV = 29010; Now find PMT given PV= 29010; Rate = 5%; NPER = 3; PMT = 10653 Ans. [d]. 559. Need based method for the life cover : 52.42 Lakhs PMT = 3 Lakhs; NPER= 40; Rate = 3%; PV = 71.42 Lakhs Add final cost [1] and deduct investments [20] = 52.42 Lakhs 100 Income Replacement = 2,50,000 = 83.33 3 Lakhs 20 + 1 = 65.33 Ans. [c]. 560. Find FV given NPER = 15; Rate = 5 %; PMT = 23000 As one pays premium as advance this is Type =1 FV = 521122 Ans. [b].

2 = 121024 is PMT; Rate 8%; 3 NPER = 20; PV = 1283290


Again find PV given FV = 1283290; Rate 8%; NPER = 20; PV = 275328 Ans. [c].

552. Find PMT given FV = 1283290; Rate = 8%; NPER = 20 and Type 1. Ans. [b]. 553. Refer Chapter 11 of the Concept Book. Ans. [c]. 554. Paid up value = 120000

21 = 50400 50

bonus 70 120 10 = 84000 75 Accumulated value = 134400 = 100800 is SV. . 100 Ans. [e]. 555. 33.10 2 3 + 4 = 32.1 1500 = 48150. Ans. [b]. 556. PMT= 6.8 Lak hs; Rate = 10%; NPER = 35; PV = 65.58 Lakhs (HLV); GAP = PV 20 Lakhs = 46 Lakhs. Ans. [a].

Workbook

Risk Management and Insurance Planning

81

Notes

82

Risk Management and Insurance Planning

Workbook

Notes

Workbook

Risk Management and Insurance Planning

83

Notes

84

Risk Management and Insurance Planning

Workbook

Vous aimerez peut-être aussi